FAA Written Exam Test Preparation for PVT - Pilots

Question Bank - Private Pilot Knowledge Test

3001[B]

With respect to the certification of airmen, which is a category of aircraft?
  1. Gyroplane, helicopter, airship, free balloon.
  2. Airplane, rotorcraft, glider, lighter-than-air.
  3. Single-engine land and sea, multiengine land and sea.

3002[A]

With respect to the certification of airmen, which is a class of aircraft

  1. Single-engine land and sea, multiengine land and sea.
  2. Lighter-than-air, airship, hot air balloon, gas balloon.
  3. Airplane, rotorcraft, glider, lighter-than-air.
3003[C]

With respect to the certification of aircraft, which is a category of aircraft?

  1. Airplane, rotorcraft, glider.
  2. Landplane, seaplane.
  3. Normal, utility, acrobatic.
3004[A]

With respect to the certification of aircraft, which is a class of aircraft

  1. Airplane, rotorcraft, glider, balloon.
  2. Transport, restricted, provisional.
  3. Normal, utility, acrobatic, limited.
3005[A]

The definition of nighttime is

  1. the time between the end of evening civil twilight and the beginning of morning civil twilight.
  2. sunset to sunrise.
  3. 1 hour after sunset to 1 hour before sunrise.



3006[C]

Which V-speed represents maneuvering speed

  1. VLO.
  2. VNE.
  3. VA.
3007[A]

Which V-speed represents maximum flap extended speed?

  1. VFE.
  2. VLOF.
  3. VFC.
3008[A]

Which V-speed represents maximum landing gear extended speed?

  1. VLE.
  2. VLO.
  3. VFE.
3009[B]

VNO is defined as the

  1. normal operating range.
  2. maximum structural cruising speed.
  3. never-exceed speed.
3010[C]

VSO is defined as the

  1. stalling speed or minimum steady flight speed in a specified configuration.
  2. stalling speed or minimum takeoff safety speed.
  3. stalling speed or minimum steady flight speed in the landing configuration.
3011[B]

Which would provide the greatest gain in altitude in the shortest distance during climb after takeoff?

  1. VY.
  2. VX.
  3. VA.

3012[C]

After takeoff, which airspeed would the pilot use to gain the most altitude in a given period of time?

  1. VA.
  2. VX.
  3. VY.

3013[B]

Preventive maintenance has been performed on an aircraft. What paperwork is required?

The date the work was completed, and the name of the person who did the work must be entered in the airframe and engine logbook.

The signature, certificate number, and kind of certificate held by the person approving the work and a description of the work must be entered in the aircraft maintenance records.
A full, detailed description of the work done must be entered in the airframe logbook.
3014[C]

Which operation would be described as preventive maintenance?

Alteration of main seat support brackets.
Engine adjustments to allow automotive gas to be used.
Servicing landing gear wheel bearings.
3015[A]

Which operation would be described as preventive maintenance?

Replenishing hydraulic fluid.
Repair of landing gear brace struts.
Repair of portions of skin sheets by making additional seams.
3017[B]

When must a current pilot certificate be in the pilot's personal possession or readily accessible in the aircraft?

Only when passengers are carried.
Anytime when acting as pilot in command or as a required crewmember.
When acting as a crew chief during launch and recovery.
3019[C]

Each person who holds a pilot certificate or a medical certificate shall present it for inspection upon the request of the Administrator, the National Transportation Safety Board, or any

person in a position of authority.
authorized representative of the Department of Transportation.
federal, state, or local law enforcement officer.
3021[A]

A Third-Class Medical Certificate is issued to a 51-year-old pilot on May 3, this year. To exercise the privileges of a Private Pilot Certificate, the medical certificate will be valid until midnight on

May 31, 2 years later.
May 31, 1 year later.
May 3, 1 year later.
3022[B]

For private pilot operations, a Second-Class Medical Certificate issued to a 42-year-old pilot on July 15, this year, will expire at midnight on

July 31, 1 year later.
July 31, 2 years later.
July 15, 2 years later.
3024[C]

The pilot in command is required to hold a type rating in which aircraft?

Aircraft involved in ferry flights, training flights, or test flights.
Aircraft operated under an authorization issued by the Administrator.
Aircraft having a gross weight of more than 12,500 pounds.
3025[C]

What is the definition of a high-performance airplane?

An airplane with a normal cruise speed in excess of 200 knots.
An airplane with 180 horsepower, or retractable landing gear, flaps, and a fixed-pitch propeller.
An airplane with an engine of more than 200 horsepower.
3026[A]

Before a person holding a private pilot certificate may act as pilot in command of a high-performance airplane, that person must have

received ground and flight instruction from an authorized flight instructor who then endorses that person's logbook.
an endorsement in that person's logbook that he or she is competent to act as pilot in command.
passed a flight test in that airplane from an FAA inspector.
3027[A]

In order to act as pilot in command of a high-performance airplane, a pilot must have

received and logged ground and flight instruction in an airplane that has more than 200 horsepower.
made and logged three solo takeoffs and landings in a high-performance airplane.
passed a flight test in a high-performance airplane.
3028[A]

To act as pilot in command of an aircraft carrying passengers, a pilot must show by logbook endorsement the satisfactory completion of a flight review or completion of a pilot proficiency check within the preceding

24 calendar months.
6 calendar months.
12 calendar months.
3029[A]

If recency of experience requirements for night flight are not met and official sunset is 1830, the latest time passengers may be carried is

1929.
1859.
1829.
3030[B]

To act as pilot in command of an aircraft carrying passengers, the pilot must have made at least three takeoffs and three landings in an aircraft of the same category, class, and if a type rating is required, of the same type, within the preceding

24 calendar months.
90 days.
12 calendar months.
3031[B]

To act as pilot in command of an aircraft carrying passengers, the pilot must have made three takeoffs and three landings within the preceding 90 days in an aircraft of the same

make and model.
category, class, and type, if a type rating is required.
category and class, but not type.
3032[B]

The takeoffs and landings required to meet the recency of experience requirements for carrying passengers in a tailwheel airplane

may be touch and go or full stop.
must be to a full stop.
must be touch and go.
3033[B]

The three takeoffs and landings that are required to act as pilot in command at night must be done during the time period from

sunset to sunrise.
1 hour after sunset to 1 hour before sunrise.
the end of evening civil twilight to the beginning of morning civil twilight.
3034[C]

To meet the recency of experience requirements to act as pilot in command carrying passengers at night, a pilot must have made at least three takeoffs and three landings to a full stop within the preceding 90 days in

any aircraft.
the same type of aircraft to be used.
the same category and class of aircraft to be used.
3035[A]

If a certificated pilot changes permanent mailing address and fails to notify the FAA Airmen Certification Branch of the new address, the pilot is entitled to exercise the privileges of the pilot certificate for a period of only

30 days after the date of the move.
90 days after the date of the move.
60 days after the date of the move.
3036[B]

A certificated private pilot may not act as pilot in command of an aircraft towing a glider unless there is entered in the pilot's logbook a minimum of

100 hours of pilot flight time in any aircraft, that the pilot is using to tow a glider.
100 hours of pilot-in-command time in the aircraft category, class, and type, if required, that the pilot is using to tow a glider.
200 hours of pilot-in-command time in the aircraft category, class, and type, if required, that the pilot is using to tow a glider.
3037[A]

To act as pilot in command of an aircraft towing a glider, a pilot is required to have made within the preceding 12 months

at least three actual or simulated glider tows while accompanied by a qualified pilot.
at least three flights as observer in a glider being towed by an aircraft.
at least three flights in a powered glider.
3041[A]

Each recreational or private pilot is required to have

a biennial flight review.
an annual flight review.
a semiannual flight review.
3042[A]

If a recreational or private pilot had a flight review on August 8, this year, when is the next flight review required

August 31, 2 years later.
August 8, next year.
August 31, 1 year later.
3043[C]

How many passengers is a recreational pilot allowed to carry on board

Three.
Two.
One.
3044[B]

According to regulations pertaining to privileges and limitations, a recreational pilot may

be paid for the operating expenses of a flight.
not pay less than the pro rata share of the operating expenses of a flight with a passenger.
not be paid in any manner for the operating expenses of a flight.
3045[C]

In regard to privileges and limitations, a recreational pilot may

fly for compensation or hire within 50 nautical miles from the departure airport with a logbook endorsement.
not be paid in any manner for the operating expenses of a flight from a passenger.
not pay less than the pro rata share of the operating expenses of a flight with a passenger.
3047[A]

A recreational pilot may act as pilot in command of an aircraft that is certificated for a maximum of how many occupants

Four.
Three.
Two.
3048[A]

A recreational pilot may act as pilot in command of an aircraft with a maximum engine horsepower of
180.
200.
160.
3049[B]

What exception, if any, permits a recreational pilot to act as pilot in command of an aircraft carrying a passenger for hire

If a donation is made to a charitable organization for the flight.
There is no exception.
If the passenger pays no more than the operating expenses.
3050[B]

May a recreational pilot act as pilot in command of an aircraft in furtherance of a business

Yes, if the flight is only incidental to that business.
No, it is not allowed.
Yes, providing the aircraft does not carry a person or property for compensation or hire.
3051[C]

With respect to daylight hours, what is the earliest time a recreational pilot may take off

At the beginning of morning civil twilight.
One hour before sunrise.
At sunrise.
3052[B]

If sunset is 2021 and the end of evening civil twilight is 2043, when must a recreational pilot terminate the flight

2043.
2021.
2121.
3053[C]

When may a recreational pilot operate to or from an airport that lies within Class C airspace

Anytime the control tower is in operation.
When the ceiling is at least 1,000 feet and the surface visibility is at least 3 miles.
For the purpose of obtaining an additional certificate or rating while under the supervision of an authorized flight instructor.
3054[B]

Under what conditions may a recreational pilot operate at an airport that lies within Class D airspace and that has a part-time control tower in operation

Between sunrise and sunset when the tower is in operation, the ceiling is at least 2,500 feet, and the visibility is at least 3 miles.
Between sunrise and sunset when the tower is closed, the ceiling is at least 1,000 feet, and the visibility is at least 3 miles.
Any time when the tower is in operation, the ceiling is at least 3,000 feet, and the visibility is more than 1 mile.
3055[C]

When may a recreational pilot fly above 10,000 feet MSL

When 2,500 feet AGL or below.
When outside of controlled airspace.
When 2,000 feet AGL or below.
3056[B]

During daytime, what is the minimum flight or surface visibility required for recreational pilots in Class G airspace below 10,000 feet MSL

1 mile.
3 miles.
5 miles.
3057[B]

During daytime, what is the minimum flight visibility required for recreational pilots in controlled airspace below 10,000 feet MSL

1 mile.
3 miles.
5 miles.
3058[C]

Under what conditions, if any, may a recreational pilot demonstrate an aircraft in flight to a prospective buyer

The flight is not outside the United States.
The buyer pays all the operating expenses.
None.
3059[B]

When, if ever, may a recreational pilot act as pilot in command in an aircraft towing a banner

If the pilot has an endorsement in his/her pilot logbook from an authorized flight instructor.
It is not allowed.
If the pilot has logged 100 hours of flight time in powered aircraft.
3060[B]

When must a recreational pilot have a pilot-in-command flight check

Every 180 days.
If the pilot has less than 400 total flight hours and has not flown as pilot in command in an aircraft within the preceding 180 days.
Every 400 hours.
3061[C]

A recreational pilot may fly as sole occupant of an aircraft at night while under the supervision of a flight instructor provided the flight or surface visibility is at least

4 miles.
3 miles.
5 miles.
3066[B]

What exception, if any, permits a private pilot to act as pilot in command of an aircraft carrying passengers who pay for the flight

There is no exception.
If a donation is made to a charitable organization for the flight.
If the passengers pay all the operating expenses.
3067[B]

The width of a Federal Airway from either side of the centerline is

6 nautical miles.
4 nautical miles.
8 nautical miles.
3068[A]

Unless otherwise specified, Federal Airways include that Class E airspace extending upward from

1,200 feet above the surface up to and including 17,999 feet MSL.
the surface up to and including 18,000 feet MSL.
700 feet above the surface up to and including 17,999 feet MSL.
3069[A]

Normal VFR operations in Class D airspace with an operating control tower require the ceiling and visibility to be at least

1,000 feet and 3 miles.
2,500 feet and 3 miles.
1,000 feet and 1 mile.
3070[A]

The final authority as to the operation of an aircraft is the

pilot in command.
aircraft manufacturer.
Federal Aviation Administration.
3072[A]

If an in-flight emergency requires immediate action, the pilot in command may

deviate from the FAR's to the extent required to meet that emergency.
deviate from the FAR's to the extent required to meet the emergency, but must submit a written report to the Administrator within 24 hours.
not deviate from the FAR's unless prior to the deviation approval is granted by the Administrator.
3073[C]

When must a pilot who deviates from a regulation during an emergency send a written report of that deviation to the Administrator

Within 10 days.
Within 7 days.
Upon request.
3074[A]

Who is responsible for determining if an aircraft is in condition for safe flight

The pilot in command.
The owner or operator.
A certificated aircraft mechanic.
3075[A]

Where may an aircraft's operating limitations be found

In the current, FAA-approved flight manual, approved manual material, markings, and placards, or any combination thereof.
On the Airworthiness Certificate.
In the aircraft airframe and engine logbooks.
3076[A]

Under what conditions may objects be dropped from an aircraft

If precautions are taken to avoid injury or damage to persons or property on the surface.
If prior permission is received from the Federal Aviation Administration.
Only in an emergency.
3077[B]

A person may not act as a crewmember of a civil aircraft if alcoholic beverages have been consumed by that person within the preceding

12 hours.
8 hours.
24 hours.
3078[A]

Under what condition, if any, may a pilot allow a person who is obviously under the influence of drugs to be carried aboard an aircraft

In an emergency or if the person is a medical patient under proper care.
Only if the person does not have access to the cockpit or pilot's compartment.
Under no condition.
3079[C]

No person may attempt to act as a crewmember of a civil aircraft with

.008 percent by weight or more alcohol in the blood.
.004 percent by weight or more alcohol in the blood.
.04 percent by weight or more alcohol in the blood.
3080[B]

Which preflight action is specifically required of the pilot prior to each flight

Review wake turbulence avoidance procedures.
Become familiar with all available information concerning the flight.
Check the aircraft logbooks for appropriate entries.
3081[B]

Preflight action, as required for all flights away from the vicinity of an airport, shall include

a study of arrival procedures at airports/ heliports of intended use.
an alternate course of action if the flight cannot be completed as planned.
the designation of an alternate airport.
3082[C]

In addition to other preflight actions for a VFR flight away from the vicinity of the departure airport, regulations specifically require the pilot in command to

check the accuracy of the navigation equipment and the emergency locator transmitter (ELT).
review traffic control light signal procedures.
determine runway lengths at airports of intended use and the aircraft's takeoff and landing distance data.
3083[B]

Flight crewmembers are required to keep their safety belts and shoulder harnesses fastened during

flight in turbulent air.
takeoffs and landings.
all flight conditions.
3084[A]

Which best describes the flight conditions under which flight crewmembers are specifically required to keep their safety belts and shoulder harnesses fastened

Safety belts during takeoff and landing and while en route; shoulder harnesses during takeoff and landing.
Safety belts during takeoff and landing; shoulder harnesses during takeoff and landing and while en route.
Safety belts during takeoff and landing; shoulder harnesses during takeoff and landing.
3085[B]

With respect to passengers, what obligation, if any, does a pilot in command have concerning the use of safety belts

The pilot in command has no obligation in regard to passengers' use of safety belts.
The pilot in command must brief the passengers on the use of safety belts and notify them to fasten their safety belts during taxi, takeoff, and landing.
The pilot in command must instruct the passengers to keep their safety belts fastened for the entire flight.
3086[C]

With certain exceptions, safety belts are required to be secured about passengers during

all flight conditions.
flight in turbulent air.
taxi, takeoffs, and landings.
3087[B]

Safety belts are required to be properly secured about which persons in an aircraft and when

Pilots only, during takeoffs and landings.
Passengers, during taxi, takeoffs, and landings only.
Each person on board the aircraft during the entire flight.
3088[C]

No person may operate an aircraft in formation flight

in Class D airspace under special VFR.
over a densely populated area.
except by prior arrangement with the pilot in command of each aircraft.
3089[B]

Which aircraft has the right-of-way over all other air traffic

An aircraft on final approach to land.
An aircraft in distress.
A balloon.
3090[A]

What action is required when two aircraft of the same category converge, but not head-on

The aircraft on the left shall give way.
Each aircraft shall give way to the right.
The faster aircraft shall give way.
3091[A]

Which aircraft has the right-of-way over the other aircraft listed

Glider.
Aircraft refueling other aircraft.
Airship.
3092[C]

An airplane and an airship are converging. If the airship is left of the airplane's position, which aircraft has the right-of-way

The airplane.
Each pilot should alter course to the right.
The airship.
3093[B]

Which aircraft has the right-of-way over the other aircraft listed

Airship.
Aircraft towing other aircraft.
Gyroplane.
3094[A]

What action should the pilots of a glider and an airplane take if on a head-on collision course

Both pilots should give way to the right.
The glider pilot should give way to the right.
The airplane pilot should give way to the left.
3095[C]

When two or more aircraft are approaching an airport for the purpose of landing, the right-of-way belongs to the aircraft

that has the other to its right.
that is the least maneuverable.
at the lower altitude, but it shall not take advantage of this rule to cut in front of or to overtake another.
3096[A]

A seaplane and a motorboat are on crossing courses. If the motorboat is to the left of the seaplane, which has the right-of-way

The seaplane.
The motorboat.
Both should alter course to the right.
3097[B]

Unless otherwise authorized, what is the maximum indicated airspeed at which a person may operate an aircraft below 10,000 feet MSL

200 knots.
250 knots.
288 knots.
3098[B]

Unless otherwise authorized, the maximum indicated airspeed at which aircraft may be flown when at or below 2,500 feet AGL and within 4 nautical miles of the primary airport of Class C airspace is

200 knots.
250 knots.
230 knots.
3099[B]

When flying in the airspace underlying Class B airspace, the maximum speed authorized is

230 knots.
200 knots.
250 knots.
3100[A]

When flying in a VFR corridor designated through Class B airspace, the maximum speed authorized is

200 knots.
250 knots.
180 knots.
3101[A]

Except when necessary for takeoff or landing, what is the minimum safe altitude for a pilot to operate an aircraft anywhere

An altitude allowing, if a power unit fails, an emergency landing without undue hazard to persons or property on the surface.
An altitude of 500 feet above the surface and no closer than 500 feet to any person, vessel, vehicle, or structure.
An altitude of 500 feet above the highest obstacle within a horizontal radius of 1,000 feet.
3102[C]

Except when necessary for takeoff or landing, what is the minimum safe altitude required for a pilot to operate an aircraft over congested areas

An altitude of 1,000 feet above any person, vessel, vehicle, or structure.
An altitude of 500 feet above the highest obstacle within a horizontal radius of 1,000 feet of the aircraft.
An altitude of 1,000 feet above the highest obstacle within a horizontal radius of 2,000 feet of the aircraft.
3103[C]

Except when necessary for takeoff or landing, what is the minimum safe altitude required for a pilot to operate an aircraft over other than a congested area

An altitude of 500 feet above the highest obstacle within a horizontal radius of 1,000 feet.
An altitude allowing, if a power unit fails, an emergency landing without undue hazard to persons or property on the surface.
An altitude of 500 feet AGL, except over open water or a sparsely populated area, which requires 500 feet from any person, vessel, vehicle, or structure.
3104[A]

Except when necessary for takeoff or landing, an aircraft may not be operated closer than what distance from any person, vessel, vehicle, or structure

500 feet.
700 feet.
1,000 feet.
3105[C]

If an altimeter setting is not available before flight, to which altitude should the pilot adjust the altimeter

Pressure altitude corrected for nonstandard temperature.
The elevation of the nearest airport corrected to mean sea level.
The elevation of the departure area.
3106[A]

Prior to takeoff, the altimeter should be set to which altitude or altimeter setting

The current local altimeter setting, if available, or the departure airport elevation.
The corrected density altitude of the departure airport.
The corrected pressure altitude for the departure airport.
3107[B]

At what altitude shall the altimeter be set to 29.92, when climbing to cruising flight level

24,000 feet MSL.
18,000 feet MSL.
14,500 feet MSL.
3108[A]

When an ATC clearance has been obtained, no pilot in command may deviate from that clearance, unless that pilot obtains an amended clearance. The one exception to this regulation is

an emergency.
when the clearance states "at pilot's discretion.''
if the clearance contains a restriction.
3109[C]

When would a pilot be required to submit a detailed report of an emergency which caused the pilot to deviate from an ATC clearance

Immediately.
Within 7 days.
When requested by ATC.
3110[A]

What action, if any, is appropriate if the pilot deviates from an ATC instruction during an emergency and is given priority

File a detailed report within 48 hours to the chief of the appropriate ATC facility, if requested.
File a report to the FAA Administrator, as soon as possible.
Take no special action since you are pilot in command.
3111[C]

A steady green light signal directed from the control tower to an aircraft in flight is a signal that the pilot

should give way to other aircraft and continue circling.
should return for landing.
is cleared to land.
3112[A]

Which light signal from the control tower clears a pilot to taxi

Flashing green.
Flashing white.
Steady green.
3113[A]

If the control tower uses a light signal to direct a pilot to give way to other aircraft and continue circling, the light will be

steady red.
flashing red.
alternating red and green.
3114[C]

A flashing white light signal from the control tower to a taxiing aircraft is an indication to

taxi only on taxiways and not cross runways.
taxi at a faster speed.
return to the starting point on the airport.
3115[A]

An alternating red and green light signal directed from the control tower to an aircraft in flight is a signal to

exercise extreme caution.
hold position.
not land; the airport is unsafe.
3116[A]

While on final approach for landing, an alternating green and red light followed by a flashing red light is received from the control tower. Under these circumstances, the pilot should

exercise extreme caution and abandon the approach, realizing the airport is unsafe for landing.
abandon the approach, circle the airport to the right, and expect a flashing white light when the airport is safe for landing.
discontinue the approach, fly the same traffic pattern and approach again, and land.
3117[C]

A blue segmented circle on a Sectional Chart depicts which class airspace

Class B.
Class C.
Class D.
3118[A]

Airspace at an airport with a part-time control tower is classified as Class D airspace only

when the associated control tower is in operation.
when the associated Flight Service Station is in operation.
when the weather minimums are below basic VFR.
3119[B]

Unless otherwise authorized, two-way radio communications with Air Traffic Control are required for landings or takeoffs

at all tower controlled airports only when weather conditions are less than VFR.
at all tower controlled airports regardless of weather conditions.
at all tower controlled airports within Class D airspace only when weather conditions are less than VFR.
3120[B]

Each pilot of an aircraft approaching to land on a runway served by a visual approach slope indicator (VASI) shall

maintain a 3° glide to the runway.
maintain an altitude at or above the glide slope.
stay high until the runway can be reached in a power-off landing.
3121[A]

When approaching to land on a runway served by a visual approach slope indicator (VASI), the pilot shall

maintain an altitude at or above the glide slope.
maintain an altitude that captures the glide slope at least 2 miles downwind from the runway threshold.
remain on the glide slope and land between the two-light bar.
3122[B]

Which is appropriate for a helicopter approaching an airport for landing

Fly right-hand traffic.
Avoid the flow of fixed-wing traffic.
Remain below the airplane traffic pattern altitude.
3123[B]

Which is the correct traffic pattern departure procedure to use at a noncontrolled airport

Make all turns to the left.
Comply with any FAA traffic pattern established for the airport.
Depart in any direction consistent with safety, after crossing the airport boundary.
3124[B]

Two-way radio communication must be established with the Air Traffic Control facility having jurisdiction over the area prior to entering which class airspace

Class E.
Class C.
Class G.
3125[A]

What minimum radio equipment is required for operation within Class C airspace

Two-way radio communications equipment, a 4096-code transponder, and an encoding altimeter.
Two-way radio communications equipment and a 4096-code transponder.
Two-way radio communications equipment, a 4096-code transponder, and DME.
3126[B]

What minimum pilot certification is required for operation within Class B airspace

Private Pilot Certificate with an instrument rating.
Private Pilot Certificate or Student Pilot Certificate with appropriate logbook endorsements.
Recreational Pilot Certificate.
3127[B]

What minimum pilot certification is required for operation within Class B airspace

Private Pilot Certificate with an instrument rating.
Private Pilot Certificate or Student Pilot Certificate with appropriate logbook endorsements.
Commercial Pilot Certificate.
3128[B]

What minimum radio equipment is required for VFR operation within Class B airspace

Two-way radio communications equipment, a 4096-code transponder, an encoding altimeter, and a VOR or TACAN receiver.
Two-way radio communications equipment, a 4096-code transponder, and an encoding altimeter.
Two-way radio communications equipment and a 4096-code transponder.


3129[A]

An operable 4096-code transponder and Mode C encoding altimeter are required in

Class B airspace and within 30 miles of the Class B primary airport.
Class D airspace.
Class E airspace below 10,000 feet MSL.
3130[A]

In which type of airspace are VFR flights prohibited

Class A.
Class B.
Class C.
3131[B]

What is the specific fuel requirement for flight under VFR during daylight hours in an airplane

Enough to fly to the first point of intended landing and to fly after that for 45 minutes at normal cruising speed.
Enough to fly to the first point of intended landing and to fly after that for 30 minutes at normal cruising speed.
Enough to complete the flight at normal cruising speed with adverse wind conditions.
3132[B]

What is the specific fuel requirement for flight under VFR at night in an airplane

Enough to complete the flight at normal cruising speed with adverse wind conditions.
Enough to fly to the first point of intended landing and to fly after that for 45 minutes at normal cruising speed.
Enough to fly to the first point of intended landing and to fly after that for 30 minutes at normal cruising speed.
3134[B]

What minimum visibility and clearance from clouds are required for a recreational pilot in Class G airspace at 1,200 feet AGL or below during daylight hours

3 miles visibility, 500 feet below the clouds.
3 miles visibility and clear of clouds.
1 mile visibility and clear of clouds.
3135[B]

Outside controlled airspace, the minimum flight visibility requirement for a recreational pilot flying VFR above 1,200 feet AGL and below 10,000 feet MSL during daylight hours is

1 mile.
3 miles.
5 miles.
3136[A]

During operations within controlled airspace at altitudes of less than 1,200 feet AGL, the minimum horizontal distance from clouds requirement for VFR flight is

2,000 feet.
1,000 feet.
1,500 feet.
3137[B]

What minimum visibility and clearance from clouds are required for VFR operations in Class G airspace at 700 feet AGL or below during daylight hours

3 miles visibility and clear of clouds.
1 mile visibility and clear of clouds.
1 mile visibility, 500 feet below, 1,000 feet above, and 2,000 feet horizontal clearance from clouds.
3138[C]

What minimum flight visibility is required for VFR flight operations on an airway below 10,000 feet MSL

1 mile.
4 miles.
3 miles.
3139[B]

The minimum distance from clouds required for VFR operations on an airway below 10,000 feet MSL is

remain clear of clouds.
500 feet below, 1,000 feet above, and 2,000 feet horizontally.
500 feet above, 1,000 feet below, and 2,000 feet horizontally.
3140[B]

During operations within controlled airspace at altitudes of more than 1,200 feet AGL, but less than 10,000 feet MSL, the minimum distance above clouds requirement for VFR flight is

500 feet.
1,000 feet.
1,500 feet.
3141[A]

VFR flight in controlled airspace above 1,200 feet AGL and below 10,000 feet MSL requires a minimum visibility and vertical cloud clearance of

3 miles, and 500 feet below or 1,000 feet above the clouds in controlled airspace.
5 miles, and 1,000 feet below or 1,000 feet above the clouds at all altitudes.
5 miles, and 1,000 feet below or 1,000 feet above the clouds only in Class A airspace.
3142[B]

During operations outside controlled airspace at altitudes of more than 1,200 feet AGL, but less than 10,000 feet MSL, the minimum flight visibility for VFR flight at night is

1 mile.
3 miles.
5 miles.
3143[C]

Outside controlled airspace, the minimum flight visibility requirement for VFR flight above 1,200 feet AGL and below 10,000 feet MSL during daylight hours is

5 miles.
3 miles.
1 mile.
3144[A]

During operations outside controlled airspace at altitudes of more than 1,200 feet AGL, but less than 10,000 feet MSL, the minimum distance below clouds requirement for VFR flight at night is

500 feet.
1,000 feet.
1,500 feet.
3145[A]

The minimum flight visibility required for VFR flights above 10,000 feet MSL and more than 1,200 feet AGL in controlled airspace is

5 miles.
3 miles.
1 mile.
3146[C]

For VFR flight operations above 10,000 feet MSL and more than 1,200 feet AGL, the minimum horizontal distance from clouds required is

2,000 feet.
1,000 feet.
1 mile.
3147[A]

During operations at altitudes of more than 1,200 feet AGL and at or above 10,000 feet MSL, the minimum distance above clouds requirement for VFR flight is

1,000 feet.
500 feet.
1,500 feet.
3148[C]

No person may take off or land an aircraft under basic VFR at an airport that lies within Class D airspace unless the

flight visibility at that airport is at least 1 mile.
ground visibility at that airport is at least 1 mile.
ground visibility at that airport is at least 3 miles.
3149[A]

The basic VFR weather minimums for operating an aircraft within Class D airspace are

1,000-foot ceiling and 3 miles visibility.
500-foot ceiling and 1 mile visibility.
clear of clouds and 2 miles visibility.
3150[A]

A special VFR clearance authorizes the pilot of an aircraft to operate VFR while within Class D airspace when the visibility is

at least 1 mile and the aircraft can remain clear of clouds.
at least 3 miles and the aircraft can remain clear of clouds.
less than 1 mile and the ceiling is less than 1,000 feet.
3151[C]

What is the minimum weather condition required for airplanes operating under special VFR in Class D airspace

1 mile flight visibility and 1,000-foot ceiling.
3 miles flight visibility and 1,000-foot ceiling.
1 mile flight visibility.
3153[C]

What are the minimum requirements for airplane operations under special VFR in Class D airspace at night

The airplane must be under radar surveillance at all times while in Class D airspace.
The airplane must be equipped for IFR with an altitude reporting transponder.
The pilot must be instrument rated, and the airplane must be IFR equipped.
3154[C]

No person may operate an airplane within Class D airspace at night under special VFR unless the

flight can be conducted 500 feet below the clouds.
flight visibility is at least 3 miles.
airplane is equipped for instrument flight.
3155[C]

Which cruising altitude is appropriate for a VFR flight on a magnetic course of 135°

Even thousandths plus 500 feet.
Even thousandths.
Odd thousandths plus 500 feet.
3156[C]

Which VFR cruising altitude is acceptable for a flight on a Victor Airway with a magnetic course of 175°? The terrain is less than 1,000 feet.

5,000 feet.
4,500 feet.
5,500 feet.
3157[A]

Which VFR cruising altitude is appropriate when flying above 3,000 feet AGL on a magnetic course of 185°

4,500 feet.
4,000 feet.
5,000 feet.
3158[B]

Each person operating an aircraft at a VFR cruising altitude shall maintain an odd-thousand plus 500-foot altitude while on a

true course of 0° through 179°.
magnetic course of 0° through 179°.
magnetic heading of 0° through 179°.
3159[C]

In addition to a valid Airworthiness Certificate, what documents or records must be aboard an aircraft during flight

Radio operator's permit, and repair and alteration forms.
Aircraft engine and airframe logbooks, and owner's manual.
Operating limitations and Registration Certificate.
3160[A]

When must batteries in an emergency locator transmitter (ELT) be replaced or recharged, if rechargeable

When the ELT has been in use for more than 1 cumulative hour.
When the ELT can no longer be heard over the airplane's communication radio receiver.
After any inadvertent activation of the ELT.
3161[A]

When are non-rechargeable batteries of an emergency locator transmitter (ELT) required to be replaced

When 50 percent of their useful life expires.
Every 24 months.
At the time of each 100-hour or annual inspection.
3162[C]

Except in Alaska, during what time period should lighted position lights be displayed on an aircraft

End of evening civil twilight to the beginning of morning civil twilight.
1 hour after sunset to 1 hour before sunrise.
Sunset to sunrise.
3163[A]

When operating an aircraft at cabin pressure altitudes above 12,500 feet MSL up to and including 14,000 feet MSL, supplemental oxygen shall be used during

that flight time in excess of 30 minutes at those altitudes.
that flight time in excess of 10 minutes at those altitudes.
the entire flight time at those altitudes.
3164[B]

Unless each occupant is provided with supplemental oxygen, no person may operate a civil aircraft of U.S. registry above a maximum cabin pressure altitude of

14,000 feet MSL.
15,000 feet MSL.
12,500 feet MSL.
3165[B]

An operable 4096-code transponder with an encoding altimeter is required in which airspace

Class D and Class E (below 10,000 feet MSL).
Class A, Class B (and within 30 miles of the Class B primary airport), and Class C.
Class D and Class G (below 10,000 feet MSL).
3166[A]

With certain exceptions, all aircraft within 30 miles of a Class B primary airport from the surface upward to 10,000 feet MSL must be equipped with

an operable transponder having either Mode S or 4096-code capability with Mode C automatic altitude reporting capability.
an operable VOR or TACAN receiver and an ADF receiver.
instruments and equipment required for IFR operations.
3167[B]

No person may operate an aircraft in acrobatic flight when

less than 2,500 feet AGL.
over any congested area of a city, town, or settlement.
flight visibility is less than 5 miles.
3168[B]

In which controlled airspace is acrobatic flight prohibited

All Class G airspace.
Class D airspace, Class E airspace designated for Federal Airways.
All Class E airspace below 1,500 feet AGL.
3169[B]

What is the lowest altitude permitted for acrobatic flight

2,000 feet AGL.
1,500 feet AGL.
1,000 feet AGL.
3170[A]

No person may operate an aircraft in acrobatic flight when the flight visibility is less than

3 miles.
5 miles.
7 miles.
3171[C]

A chair-type parachute must have been packed by a certificated and appropriately rated parachute rigger within the preceding

60 days.
90 days.
120 days.
3172[B]

An approved chair-type parachute may be carried in an aircraft for emergency use if it has been packed by an appropriately rated parachute rigger within the preceding

365 days.
120 days.
180 days.
3173[B]

With certain exceptions, when must each occupant of an aircraft wear an approved parachute

When intentionally banking in excess of 30°.
When intentionally pitching the nose of the aircraft up or down 30° or more.
When a door is removed from the aircraft to facilitate parachute jumpers.
3178[C]

Which is normally prohibited when operating a restricted category civil aircraft

Flight within Class D airspace.
Flight under instrument flight rules.
Flight over a densely populated area.
3179[C]

Unless otherwise specifically authorized, no person may operate an aircraft that has an experimental certificate

from the primary airport within Class D airspace.
beneath the floor of Class B airspace.
over a densely populated area or in a congested airway.
3180[C]

The responsibility for ensuring that an aircraft is maintained in an airworthy condition is primarily that of the

pilot in command.
mechanic who performs the work.
owner or operator.
3181[A]

The responsibility for ensuring that maintenance personnel make the appropriate entries in the aircraft maintenance records indicating the aircraft has been approved for return to service lies with the

owner or operator.
pilot in command.
mechanic who performed the work.
3182[B]

Completion of an annual inspection and the return of the aircraft to service should always be indicated by

the relicensing date on the Registration Certificate.
an appropriate notation in the aircraft maintenance records.
an inspection sticker placed on the instrument panel that lists the annual inspection completion date.
3183[B]

If an alteration or repair substantially affects an aircraft's operation in flight, that aircraft must be test flown by an appropriately-rated pilot and approved for return to service prior to being operated

by any private pilot.
with passengers aboard.
for compensation or hire.
3184[B]

Before passengers can be carried in an aircraft that has been altered in a manner that may have appreciably changed its flight characteristics, it must be flight tested by an appropriately-rated pilot who holds at least a

Commercial Pilot Certificate with an instrument rating.
Private Pilot Certificate.
Commercial Pilot Certificate and a mechanic's certificate.
3185[A]

An aircraft's annual inspection was performed on July 12, this year. The next annual inspection will be due no later than

July 31, next year.
July 13, next year.
July 1, next year.
3186[C]

To determine the expiration date of the last annual aircraft inspection, a person should refer to the

Airworthiness Certificate.
Registration Certificate.
aircraft maintenance records.
3187[A]

How long does the Airworthiness Certificate of an aircraft remain valid

As long as the aircraft is maintained and operated as required by Federal Aviation Regulations.
Indefinitely, unless the aircraft suffers major damage.
As long as the aircraft has a current Registration Certificate.
3188[B]

What aircraft inspections are required for rental aircraft that are also used for flight instruction

Biannual and 100-hour inspections.
Annual and 100-hour inspections.
Annual and 50-hour inspections.
3189[B]

An aircraft had a 100-hour inspection when the tachometer read 1259.6. When is the next 100-hour inspection due

1349.6 hours.
1359.6 hours.
1369.6 hours.
3190[B]

A 100-hour inspection was due at 3302.5 hours. The 100-hour inspection was actually done at 3309.5 hours. When is the next 100-hour inspection due

3312.5 hours.
3402.5 hours.
3409.5 hours.
3191[B]

No person may use an ATC transponder unless it has been tested and inspected within at least the preceding

6 calendar months.
24 calendar months.
12 calendar months.
3192[C]

Maintenance records show the last transponder inspection was performed on September 1, 1993. The next inspection will be due no later than

September 30, 1994.
September 1, 1995.
September 30, 1995.
3193[C]

Which records or documents shall the owner or operator of an aircraft keep to show compliance with an applicable Airworthiness Directive

Airworthiness and Registration Certificates.
Airworthiness Certificate and Pilot's Operating Handbook.
Aircraft maintenance records.
3194[B]

If an aircraft is involved in an accident which results in substantial damage to the aircraft, the nearest NTSB field office should be notified

within 7 days.
immediately.
within 48 hours.
3195[B]

Which incident requires an immediate notification to the nearest NTSB field office

A forced landing due to engine failure.
Flight control system malfunction or failure.
Landing gear damage, due to a hard landing.
3196[C]

Which incident would necessitate an immediate notification to the nearest NTSB field office

An in-flight generator/alternator failure.
An in-flight loss of VOR receiver capability.
An in-flight fire.
3197[A]

Which incident requires an immediate notification be made to the nearest NTSB field office

An overdue aircraft that is believed to be involved in an accident.
An in-flight generator or alternator failure.
An in-flight radio communications failure.
3198[B]

May aircraft wreckage be moved prior to the time the NTSB takes custody

Yes, but only if moved by a federal, state, or local law enforcement officer.
Yes, but only to protect the wreckage from further damage.
No, it may not be moved under any circumstances.
3199[A]

The operator of an aircraft that has been involved in an accident is required to file an accident report within how many days

10.
5.
7.
3200[A]

The operator of an aircraft that has been involved in an incident is required to submit a report to the nearest field office of the NTSB

when requested.
within 7 days.
within 10 days.
3201[A]

The four forces acting on an airplane in flight are

lift, weight, thrust, and drag.
lift, weight, gravity, and thrust.
lift, gravity, power, and friction.
3202[A]

When are the four forces that act on an airplane in equilibrium

During unaccelerated flight.
When the aircraft is at rest on the ground.
When the aircraft is accelerating.
3203[C]

(Refer to figure 1.) The acute angle A is the angle of

dihedral.
incidence.
attack.
3204[B]

The term "angle of attack'' is defined as the angle

formed by the longitudinal axis of the airplane and the chord line of the wing.
between the wing chord line and the relative wind.
between the airplane's climb angle and the horizon.
3205[B]

What is the relationship of lift, drag, thrust, and weight when the airplane is in straight-and-level flight

Lift, drag, and weight equal thrust.
Lift equals weight and thrust equals drag.
Lift and weight equal thrust and drag.
3206[A]

How will frost on the wings of an airplane affect takeoff performance

Frost will disrupt the smooth flow of air over the wing, adversely affecting its lifting capability.
Frost will change the camber of the wing, increasing its lifting capability.
Frost will cause the airplane to become airborne with a higher angle of attack, decreasing the stall speed.
3207[A]

In what flight condition is torque effect the greatest in a single-engine airplane

Low airspeed, high power, high angle of attack.
High airspeed, high power, high angle of attack.
Low airspeed, low power, low angle of attack.
3208[C]

The left turning tendency of an airplane caused by P-factor is the result of the

gyroscopic forces applied to the rotating propeller blades acting 90° in advance of the point the force was applied.
clockwise rotation of the engine and the propeller turning the airplane counter-clockwise.
propeller blade descending on the right, producing more thrust than the ascending blade on the left.
3209[A]

When does P-factor cause the airplane to yaw to the left

When at high angles of attack.
When at high airspeeds.
When at low angles of attack.
3210[A]

An airplane said to be inherently stable will

require less effort to control.
be difficult to stall.
not spin.
3211[C]

What determines the longitudinal stability of an airplane

The relationship of thrust and lift to weight and drag.
The effectiveness of the horizontal stabilizer, rudder, and rudder trim tab.
The location of the CG with respect to the center of lift.
3212[A]

What causes an airplane (except a T-tail) to pitch nosedown when power is reduced and controls are not adjusted

The downwash on the elevators from the propeller slipstream is reduced and elevator effectiveness is reduced.
The CG shifts forward when thrust and drag are reduced.
When thrust is reduced to less than weight, lift is also reduced and the wings can no longer support the weight.
3213[A]

What is the purpose of the rudder on an airplane

To control yaw.
To control overbanking tendency.
To control roll.
3214[A]

(Refer to figure 2.) If an airplane weighs 2,300 pounds, what approximate weight would the airplane structure be required to support during a 60° banked turn while maintaining altitude

4,600 pounds.
2,300 pounds.
3,400 pounds.
3215[B]

(Refer to figure 2.) If an airplane weighs 3,300 pounds, what approximate weight would the airplane structure be required to support during a 30° banked turn while maintaining altitude

1,200 pounds.
3,960 pounds.
3,100 pounds.
3216[C]

(Refer to figure 2.) If an airplane weighs 4,500 pounds, what approximate weight would the airplane structure be required to support during a 45° banked turn while maintaining altitude

4,500 pounds.
7,200 pounds.
6,750 pounds.
3217[C]

The amount of excess load that can be imposed on the wing of an airplane depends upon the

position of the CG.
abruptness at which the load is applied.
speed of the airplane.
3218[C]

Which basic flight maneuver increases the load factor on an airplane as compared to straight-and-level flight

Stalls.
Climbs.
Turns.
3219[A]

One of the main functions of flaps during approach and landing is to

increase the angle of descent without increasing the airspeed.
decrease the angle of descent without increasing the airspeed.
permit a touchdown at a higher indicated airspeed.
3220[B]

What is one purpose of wing flaps

To relieve the pilot of maintaining continuous pressure on the controls.
To enable the pilot to make steeper approaches to a landing without increasing the airspeed.
To decrease wing area to vary the lift.
3221[B]

Excessively high engine temperatures will

not appreciably affect an aircraft engine.
cause loss of power, excessive oil consumption, and possible permanent internal engine damage.
cause damage to heat-conducting hoses and warping of the cylinder cooling fins.
3222[C]

If the engine oil temperature and cylinder head temperature gauges have exceeded their normal operating range, the pilot may have been operating with

higher-than-normal oil pressure.
the mixture set too rich.
too much power and with the mixture set too lean.
3223[B]

One purpose of the dual ignition system on an aircraft engine is to provide for

balanced cylinder head pressure.
improved engine performance.
uniform heat distribution.
3225[A]

The operating principle of float-type carburetors is based on the

difference in air pressure at the venturi throat and the air inlet.
increase in air velocity in the throat of a venturi causing an increase in air pressure.
automatic metering of air at the venturi as the aircraft gains altitude.
3226[C]

The basic purpose of adjusting the fuel/air mixture at altitude is to

increase the amount of fuel in the mixture to compensate for the decrease in pressure and density of the air.
decrease the amount of fuel in the mixture in order to compensate for increased air density.
decrease the fuel flow in order to compensate for decreased air density.
3227[A]

During the run-up at a high-elevation airport, a pilot notes a slight engine roughness that is not affected by the magneto check but grows worse during the carburetor heat check. Under these circumstances, what would be the most logical initial action

Check the results obtained with a leaner setting of the mixture.
Taxi back to the flight line for a maintenance check.
Reduce manifold pressure to control detonation.
3228[A]

While cruising at 9,500 feet MSL, the fuel/air mixture is properly adjusted. What will occur if a descent to 4,500 feet MSL is made without readjusting the mixture

The fuel/air mixture may become excessively lean.
There will be more fuel in the cylinders than is needed for normal combustion, and the excess fuel will absorb heat and cool the engine.
The excessively rich mixture will create higher cylinder head temperatures and may cause detonation.
3229[A]

Which condition is most favorable to the development of carburetor icing

Temperature between 20 and 70 °F and high humidity.
Temperature between 32 and 50 °F and low humidity.
Any temperature below freezing and a relative humidity of less than 50 percent.
3230[A]

The possibility of carburetor icing exists even when the ambient air temperature is as

high as 70 °F and the relative humidity is high.
high as 95 °F and there is visible moisture.
low as 0 °F and the relative humidity is high.
3231[C]

If an aircraft is equipped with a fixed-pitch propeller and a float-type carburetor, the first indication of carburetor ice would most likely be

engine roughness.
a drop in oil temperature and cylinder head temperature.
loss of RPM.
3232[C]

Applying carburetor heat will

not affect the fuel/air mixture.
result in more air going through the carburetor.
enrich the fuel/air mixture.
3233[C]

What change occurs in the fuel/air mixture when carburetor heat is applied

The fuel/air mixture becomes leaner.
A decrease in RPM results from the lean mixture.
The fuel/air mixture becomes richer.
3234[C]

Generally speaking, the use of carburetor heat tends to

have no effect on engine performance.
increase engine performance.
decrease engine performance.
3235[A]

The presence of carburetor ice in an aircraft equipped with a fixed-pitch propeller can be verified by applying carburetor heat and noting

a decrease in RPM and then a gradual increase in RPM.
a decrease in RPM and then a constant RPM indication.
an increase in RPM and then a gradual decrease in RPM.
3236[A]

With regard to carburetor ice, float-type carburetor systems in comparison to fuel injection systems are generally considered to be

more susceptible to icing.
equally susceptible to icing.
susceptible to icing only when visible moisture is present.
3237[A]

If the grade of fuel used in an aircraft engine is lower than specified for the engine, it will most likely cause

detonation.
lower cylinder head temperatures.
a mixture of fuel and air that is not uniform in all cylinders.
3238[C]

Detonation occurs in a reciprocating aircraft engine when

hot spots in the combustion chamber ignite the fuel/air mixture in advance of normal ignition.
the spark plugs are fouled or shorted out or the wiring is defective.
the unburned charge in the cylinders explodes instead of burning normally.
3239[A]

If a pilot suspects that the engine (with a fixed-pitch propeller) is detonating during climb-out after takeoff, the initial corrective action to take would be to

lower the nose slightly to increase airspeed.
lean the mixture.
apply carburetor heat.
3240[A]

The uncontrolled firing of the fuel/air charge in advance of normal spark ignition is known as

pre-ignition.
detonation.
combustion.
3241[A]

Which would most likely cause the cylinder head temperature and engine oil temperature gauges to exceed their normal operating ranges

Using fuel that has a lower-than-specified fuel rating.
Using fuel that has a higher-than-specified fuel rating.
Operating with higher-than-normal oil pressure.
3242[B]

What type fuel can be substituted for an aircraft if the recommended octane is not available

Unleaded automotive gas of the same octane rating.
The next higher octane aviation gas.
The next lower octane aviation gas.
3243[A]

Filling the fuel tanks after the last flight of the day is considered a good operating procedure because this will

prevent moisture condensation by eliminating airspace in the tanks.
force any existing water to the top of the tank away from the fuel lines to the engine.
prevent expansion of the fuel by eliminating airspace in the tanks.
3244[B]

For internal cooling, reciprocating aircraft engines are especially dependent on

a properly functioning thermostat.
the circulation of lubricating oil.
air flowing over the exhaust manifold.
3245[B]

An abnormally high engine oil temperature indication may be caused by

operating with a too high viscosity oil.
the oil level being too low.
operating with an excessively rich mixture.
3246[B]

What effect does high density altitude, as compared to low density altitude, have on propeller efficiency and why

Efficiency is reduced due to the increased force of the propeller in the thinner air.
Efficiency is reduced because the propeller exerts less force at high density altitudes than at low density altitudes.
Efficiency is increased due to less friction on the propeller blades.
3247[A]

If the pitot tube and outside static vents become clogged, which instruments would be affected

The altimeter, airspeed indicator, and vertical speed indicator.
The altimeter, airspeed indicator, and turn-and-slip indicator.
The altimeter, attitude indicator, and turn-and-slip indicator.
3248[A]

Which instrument will become inoperative if the pitot tube becomes clogged

Airspeed.
Vertical speed.
Altimeter.
3249[A]

Which instrument(s) will become inoperative if the static vents become clogged

Airspeed, altimeter, and vertical speed.
Altimeter only.
Airspeed only.
3250[A]

(Refer to figure 3.) Altimeter 1 indicates

10,500 feet.
1,500 feet.
500 feet.
3251[A]

(Refer to figure 3.) Altimeter 2 indicates

14,500 feet.
1,500 feet.
4,500 feet.
3252[C]

(Refer to figure 3.) Altimeter 3 indicates

15,940 feet.
10,950 feet.
9,500 feet.
3253[B]

(Refer to figure 3.) Which altimeter(s) indicate(s) more than 10,000 feet

1 only.
1 and 2 only.
1, 2, and 3.
3254[C]

Altimeter setting is the value to which the barometric pressure scale of the altimeter is set so the altimeter indicates

absolute altitude at field elevation.
calibrated altitude at field elevation.
true altitude at field elevation.
3255[A]

How do variations in temperature affect the altimeter

Pressure levels are raised on warm days and the indicated altitude is lower than true altitude.
Higher temperatures expand the pressure levels and the indicated altitude is higher than true altitude.
Lower temperatures lower the pressure levels and the indicated altitude is lower than true altitude.
3256[B]

What is true altitude

The vertical distance of the aircraft above the surface.
The vertical distance of the aircraft above sea level.
The height above the standard datum plane.
3257[B]

What is absolute altitude

The altitude read directly from the altimeter.
The vertical distance of the aircraft above the surface.
The height above the standard datum plane.
3258[C]

What is density altitude

The height above the standard datum plane.
The altitude read directly from the altimeter.
The pressure altitude corrected for nonstandard temperature.
3259[A]

What is pressure altitude

The altitude indicated when the barometric pressure scale is set to 29.92.
The indicated altitude corrected for nonstandard temperature and pressure.
The indicated altitude corrected for position and installation error.
3260[C]

Under what condition is indicated altitude the same as true altitude

If the altimeter has no mechanical error.
When at 18,000 feet MSL with the altimeter set at 29.92.
When at sea level under standard conditions.
3261[A]

If it is necessary to set the altimeter from 29.15 to 29.85, what change occurs

700-foot increase in indicated altitude.
70-foot increase in density altitude.
70-foot increase in indicated altitude.
3262[A]

The pitot system provides impact pressure for which instrument

Airspeed indicator.
Altimeter.
Vertical-speed indicator.
3263[A]

As altitude increases, the indicated airspeed at which a given airplane stalls in a particular configuration will

remain the same regardless of altitude.
decrease as the true airspeed decreases.
decrease as the true airspeed increases.
3264[C]

What does the red line on an airspeed indicator represent

Maneuvering speed.
Turbulent or rough-air speed.
Never-exceed speed.
3265[A]

(Refer to figure 4.) What is the full flap operating range for the airplane

60 to 100 MPH.
65 to 165 MPH.
60 to 208 MPH.
3266[B]

(Refer to figure 4.) What is the caution range of the airplane

100 to 165 MPH.
165 to 208 MPH.
0 to 60 MPH.
3267[B]

(Refer to figure 4.) The maximum speed at which the airplane can be operated in smooth air is

165 MPH.
208 MPH.
100 MPH.
3268[B]

(Refer to figure 4.) Which color identifies the never-exceed speed

Upper limit of the white arc.
The red radial line.
Lower limit of the yellow arc.
3269[C]

(Refer to figure 4.) Which color identifies the power-off stalling speed in a specified configuration

Upper limit of the white arc.
Upper limit of the green arc.
Lower limit of the green arc.
3270[C]

(Refer to figure 4.) What is the maximum flaps-extended speed

165 MPH.
65 MPH.
100 MPH.
3271[B]

(Refer to figure 4.) Which color identifies the normal flap operating range

The lower limit of the white arc to the upper limit of the green arc.
The white arc.
The green arc.
3272[B]

(Refer to figure 4.) Which color identifies the power-off stalling speed with wing flaps and landing gear in the landing configuration

Upper limit of the white arc.
Lower limit of the white arc.
Upper limit of the green arc.
3273[C]

(Refer to figure 4.) What is the maximum structural cruising speed

100 MPH.
208 MPH.
165 MPH.
3274[A]

What is an important airspeed limitation that is not color coded on airspeed indicators

Maneuvering speed.
Maximum structural cruising speed.
Never-exceed speed.
3275[C]

(Refer to figure 5.) A turn coordinator provides an indication of the

angle of bank up to but not exceeding 30°.
attitude of the aircraft with reference to the longitudinal axis.
movement of the aircraft about the yaw and roll axis.
3276[B]

(Refer to figure 6.) To receive accurate indications during flight from a heading indicator, the instrument must be

calibrated on a compass rose at regular intervals.
periodically realigned with the magnetic compass as the gyro precesses.
set prior to flight on a known heading.
3277[C]

(Refer to figure 7.) The proper adjustment to make on the attitude indicator during level flight is to align the

horizon bar to the miniature airplane.
horizon bar to the level-flight indication.
miniature airplane to the horizon bar.
3278[A]

(Refer to figure 7.) How should a pilot determine the direction of bank from an attitude indicator such as the one illustrated

By the relationship of the miniature airplane (C) to the deflected horizon bar (B).
By the direction of deflection of the banking scale (A).
By the direction of deflection of the horizon bar (B).
3279[B]

Deviation in a magnetic compass is caused by the

difference in the location between true north and magnetic north.
magnetic fields within the aircraft distorting the lines of magnetic force.
presence of flaws in the permanent magnets of the compass.
3280[A]

In the Northern Hemisphere, a magnetic compass will normally indicate initially a turn toward the west if

a right turn is entered from a north heading.
an aircraft is accelerated while on a north heading.
a left turn is entered from a north heading.
3281[C]

In the Northern Hemisphere, a magnetic compass will normally indicate initially a turn toward the east if

an aircraft is accelerated while on a north heading.
an aircraft is decelerated while on a south heading.
a left turn is entered from a north heading.
3282[B]

In the Northern Hemisphere, a magnetic compass will normally indicate a turn toward the north if

a left turn is entered from a west heading.
an aircraft is accelerated while on an east or west heading.
a right turn is entered from an east heading.
3283[A]

In the Northern Hemisphere, the magnetic compass will normally indicate a turn toward the south when

the aircraft is decelerated while on a west heading.
a right turn is entered from a west heading.
a left turn is entered from an east heading.
3284[A]

In the Northern Hemisphere, if an aircraft is accelerated or decelerated, the magnetic compass will normally indicate

correctly when on a north or south heading.
a turn toward the south.
a turn momentarily.
3285[B]

In the Northern Hemisphere, if a glider is accelerated or decelerated, the magnetic compass will normally indicate

a turn toward south while accelerating on a west heading.
correctly only when on a north or south heading.
a turn toward north while decelerating on an east heading.
3286[B]

During flight, when are the indications of a magnetic compass accurate

During turns if the bank does not exceed 18°.
Only in straight-and-level unaccelerated flight.
As long as the airspeed is constant.
3287[C]

An airplane has been loaded in such a manner that the CG is located aft of the aft CG limit. One undesirable flight characteristic a pilot might experience with this airplane would be

stalling at higher-than-normal airspeed.
a longer takeoff run.
difficulty in recovering from a stalled condition.
3288[C]

Loading an airplane to the most aft CG will cause the airplane to be

less stable at high speeds, but more stable at low speeds.
less stable at slow speeds, but more stable at high speeds.
less stable at all speeds.
3289[B]

If the outside air temperature (OAT) at a given altitude is warmer than standard, the density altitude is

lower than pressure altitude.
higher than pressure altitude.
equal to pressure altitude.
3290[C]

Which combination of atmospheric conditions will reduce aircraft takeoff and climb performance

High temperature, low relative humidity, and low density altitude.
Low temperature, low relative humidity, and low density altitude.
High temperature, high relative humidity, and high density altitude.
3291[C]

What effect does high density altitude have on aircraft performance

It increases takeoff performance.
It increases engine performance.
It reduces climb performance.
3292[A]

(Refer to figure 8.) What is the effect of a temperature increase from 25 to 50 °F on the density altitude if the pressure altitude remains at 5,000 feet

1,650-foot increase.
1,200-foot increase.
1,400-foot increase.
3293[A]

(Refer to figure 8.) Determine the pressure altitude with an indicated altitude of 1,380 feet MSL with an altimeter setting of 28.22 at standard temperature.

3,010 feet MSL.
2,991 feet MSL.
2,913 feet MSL.
3294[A]

(Refer to figure 8.) Determine the density altitude for these conditions:Altimeter setting 29.25Runway temperature +81 °FAirport elevation 5,250 ft MSL

8,500 feet MSL.
5,877 feet MSL.
4,600 feet MSL.
3295[B]

(Refer to figure 8.) Determine the pressure altitude at an airport that is 3,563 feet MSL with an altimeter setting of 29.96.

3,556 feet MSL.
3,527 feet MSL.
3,639 feet MSL.
3296[B]

(Refer to figure 8.) What is the effect of a temperature increase from 30 to 50 °F on the density altitude if the pressure altitude remains at 3,000 feet MSL

1,100-foot decrease.
1,300-foot increase.
900-foot increase.
3297[B]

(Refer to figure 8.) Determine the pressure altitude at an airport that is 1,386 feet MSL with an altimeter setting of 29.97.

1,451 feet MSL.
1,341 feet MSL.
1,562 feet MSL.
3298[A]

(Refer to figure 8.) Determine the density altitude for these conditions:Altimeter setting 30.35Runway temperature +25 °FAirport elevation 3,894 ft MSL

2,000 feet MSL.
2,900 feet MSL.
3,500 feet MSL.
3299[A]

(Refer to figure 8.) What is the effect of a temperature decrease and a pressure altitude increase on the density altitude from 90 °F and 1,250 feet pressure altitude to 55 °F and 1,750 feet pressure altitude

1,300-foot decrease.
1,700-foot decrease.
1,700-foot increase.
3300[B]

What effect, if any, does high humidity have on aircraft performance

It has no effect on performance.
It decreases performance.
It increases performance.
3301[C]

What force makes an airplane turn

The vertical component of lift.
Centrifugal force.
The horizontal component of lift.
3302[C]

When taxiing with strong quartering tailwinds, which aileron positions should be used

Ailerons neutral.
Aileron down on the downwind side.
Aileron down on the side from which the wind is blowing.
3303[B]

Which aileron positions should a pilot generally use when taxiing in strong quartering headwinds

Ailerons neutral.
Aileron up on the side from which the wind is blowing.
Aileron down on the side from which the wind is blowing.
3304[C]

Which wind condition would be most critical when taxiing a nosewheel equipped high-wing airplane

Quartering headwind.
Direct crosswind.
Quartering tailwind.
3305[B]

(Refer to figure 9, area A.) How should the flight controls be held while taxiing a tricycle-gear equipped airplane into a left quartering headwind

Left aileron up, elevator down.
Left aileron up, elevator neutral.
Left aileron down, elevator neutral.
3306[C]

(Refer to figure 9, area B.) How should the flight controls be held while taxiing a tailwheel airplane into a right quartering headwind

Right aileron up, elevator down.
Right aileron down, elevator neutral.
Right aileron up, elevator up.
3307[C]

(Refer to figure 9, area C.) How should the flight controls be held while taxiing a tailwheel airplane with a left quartering tailwind

Left aileron up, elevator neutral.
Left aileron down, elevator neutral.
Left aileron down, elevator down.
3308[B]

(Refer to figure 9, area C.) How should the flight controls be held while taxiing a tricycle-gear equipped airplane with a left quartering tailwind

Left aileron up, elevator neutral.
Left aileron down, elevator down.
Left aileron up, elevator down.
3309[A]

In what flight condition must an aircraft be placed in order to spin

Stalled.
Partially stalled with one wing low.
In a steep diving spiral.
3310[C]

During a spin to the left, which wing(s) is/are stalled

Neither wing is stalled.
Only the left wing is stalled.
Both wings are stalled.
3311[B]

The angle of attack at which an airplane wing stalls will

increase if the CG is moved forward.
remain the same regardless of gross weight.
change with an increase in gross weight.
3312[B]

What is ground effect

The result of the disruption of the airflow patterns about the wings of an airplane to the point where the wings will no longer support the airplane in flight.
The result of the interference of the surface of the Earth with the airflow patterns about an airplane.
The result of an alteration in airflow patterns increasing induced drag about the wings of an airplane.
3313[C]

Floating caused by the phenomenon of ground effect will be most realized during an approach to land when at

a higher-than-normal angle of attack.
twice the length of the wingspan above the surface.
less than the length of the wingspan above the surface.
3314[B]

What must a pilot be aware of as a result of ground effect

Wingtip vortices increase creating wake turbulence problems for arriving and departing aircraft.
Induced drag decreases; therefore, any excess speed at the point of flare may cause considerable floating.
A full stall landing will require less up elevator deflection than would a full stall when done free of ground effect.
3315[C]

Ground effect is most likely to result in which problem

Settling to the surface abruptly during landing.
Inability to get airborne even though airspeed is sufficient for normal takeoff needs.
Becoming airborne before reaching recommended takeoff speed.
3316[B]

During an approach to a stall, an increased load factor will cause the airplane to

have a tendency to spin.
stall at a higher airspeed.
be more difficult to control.
3317[C]

Angle of attack is defined as the angle between the chord line of an airfoil and the

pitch angle of an airfoil.
rotor plane of rotation.
direction of the relative wind.
3381[A]

Every physical process of weather is accompanied by, or is the result of, a

heat exchange.
pressure differential.
movement of air.
3382[B]

What causes variations in altimeter settings between weather reporting points

Coriolis force.
Unequal heating of the Earth's surface.
Variation of terrain elevation.
3383[B]

A temperature inversion would most likely result in which weather condition

Good visibility in the lower levels of the atmosphere and poor visibility above an inversion aloft.
An increase in temperature as altitude is increased.
Clouds with extensive vertical development above an inversion aloft.
3384[A]

The most frequent type of ground or surface-based temperature inversion is that which is produced by

terrestrial radiation on a clear, relatively still night.
warm air being lifted rapidly aloft in the vicinity of mountainous terrain.
the movement of colder air under warm air, or the movement of warm air over cold air.
3385[A]

Which weather conditions should be expected beneath a low-level temperature inversion layer when the relative humidity is high

Smooth air, poor visibility, fog, haze, or low clouds.
Light wind shear, poor visibility, haze, and light rain.
Turbulent air, poor visibility, fog, low stratus type clouds, and showery precipitation.
3386[C]

What are the standard temperature and pressure values for sea level

59 °F and 29.92 millibars.
59 °C and 1013.2 millibars.
15 °C and 29.92" Hg.
3387[B]

If a pilot changes the altimeter setting from 30.11 to 29.96, what is the approximate change in indication

Altimeter will indicate 150 feet higher.
Altimeter will indicate 150 feet lower.
Altimeter will indicate .15" Hg higher.
3388[A]

Under which condition will pressure altitude be equal to true altitude

When standard atmospheric conditions exist.
When the atmospheric pressure is 29.92" Hg.
When indicated altitude is equal to the pressure altitude.
3389[A]

Under what condition is pressure altitude and density altitude the same value

At standard temperature.
At sea level, when the temperature is 0 °F.
When the altimeter has no installation error.
3390[A]

If a flight is made from an area of low pressure into an area of high pressure without the altimeter setting being adjusted, the altimeter will indicate

lower than the actual altitude above sea level.
higher than the actual altitude above sea level.
the actual altitude above sea level.
3391[C]

If a flight is made from an area of high pressure into an area of lower pressure without the altimeter setting being adjusted, the altimeter will indicate

the actual altitude above sea level.
lower than the actual altitude above sea level.
higher than the actual altitude above sea level.
3392[C]

Under what condition will true altitude be lower than indicated altitude

When density altitude is higher than indicated altitude.
In warmer than standard air temperature.
In colder than standard air temperature.
3393[C]

Which condition would cause the altimeter to indicate a lower altitude than true altitude

Air temperature lower than standard.
Atmospheric pressure lower than standard.
Air temperature warmer than standard.
3394[B]

Which factor would tend to increase the density altitude at a given airport

An increase in barometric pressure.
An increase in ambient temperature.
A decrease in relative humidity.
3395[C]

The wind at 5,000 feet AGL is southwesterly while the surface wind is southerly. This difference in direction is primarily due to

stronger Coriolis force at the surface.
stronger pressure gradient at higher altitudes.
friction between the wind and the surface.
3397[B]

What is meant by the term "dewpoint''

The temperature at which condensation and evaporation are equal.
The temperature to which air must be cooled to become saturated.
The temperature at which dew will always form.
3398[C]

The amount of water vapor which air can hold depends on the

stability of the air.
dewpoint.
air temperature.
3399[C]

Clouds, fog, or dew will always form when

relative humidity reaches 100 percent.
water vapor is present.
water vapor condenses.
3400[A]

What are the processes by which moisture is added to unsaturated air

Evaporation and sublimation.
Heating and condensation.
Supersaturation and evaporation.
3401[C]

Which conditions result in the formation of frost

The temperature of the surrounding air is at or below freezing when small drops of moisture fall on the collecting surface.
The temperature of the collecting surface is at or below freezing when small droplets of moisture fall on the surface.
The temperature of the collecting surface is at or below the dewpoint of the adjacent air and the dewpoint is below freezing.
3402[A]

The presence of ice pellets at the surface is evidence that there

is a temperature inversion with freezing rain at a higher altitude.
are thunderstorms in the area.
has been cold frontal passage.
3403[C]

What measurement can be used to determine the stability of the atmosphere

Atmospheric pressure.
Surface temperature.
Actual lapse rate.
3404[A]

What would decrease the stability of an air mass

Warming from below.
Cooling from below.
Decrease in water vapor.
3405[A]

What is a characteristic of stable air

Stratiform clouds.
Unlimited visibility.
Cumulus clouds.
3406[A]

Moist, stable air flowing upslope can be expected to

produce stratus type clouds.
cause showers and thunderstorms.
develop convective turbulence.
3407[C]

If an unstable air mass is forced upward, what type clouds can be expected

Stratus clouds with little vertical development.
Stratus clouds with considerable associated turbulence.
Clouds with considerable vertical development and associated turbulence.
3408[C]

What feature is associated with a temperature inversion

Chinook winds on mountain slopes.
An unstable layer of air.
A stable layer of air.
3409[C]

What is the approximate base of the cumulus clouds if the surface air temperature at 1,000 feet MSL is 70 °F and the dewpoint is 48 °F

4,000 feet MSL.
5,000 feet MSL.
6,000 feet MSL.
3410[B]

At approximately what altitude above the surface would the pilot expect the base of cumuliform clouds if the surface air temperature is 82 °F and the dewpoint is 38 °F

11,000 feet AGL.
10,000 feet AGL.
9,000 feet AGL.
3412[B]

What are characteristics of a moist, unstable air mass

Poor visibility and smooth air.
Cumuliform clouds and showery precipitation.
Stratiform clouds and showery precipitation.
3413[A]

What are characteristics of unstable air

Turbulence and good surface visibility.
Turbulence and poor surface visibility.
Nimbostratus clouds and good surface visibility.
3414[C]

A stable air mass is most likely to have which characteristic

Turbulent air.
Showery precipitation.
Smooth air.
3415[A]

The suffix "nimbus,'' used in naming clouds, means

a rain cloud.
a middle cloud containing ice pellets.
a cloud with extensive vertical development.
3416[A]

Clouds are divided into four families according to their

height range.
composition.
outward shape.
3417[C]

An almond or lens-shaped cloud which appears stationary, but which may contain winds of 50 knots or more, is referred to as

a funnel cloud.
an inactive frontal cloud.
a lenticular cloud.
3418[B]

Crests of standing mountain waves may be marked by stationary, lens-shaped clouds known as

roll clouds.
standing lenticular clouds.
mammatocumulus clouds.
3419[C]

What clouds have the greatest turbulence

Towering cumulus.
Nimbostratus.
Cumulonimbus.
3420[C]

What cloud types would indicate convective turbulence

Nimbostratus clouds.
Cirrus clouds.
Towering cumulus clouds.
3421[C]

The boundary between two different air masses is referred to as a

frontogenesis.
frontolysis.
front.
3422[B]

One of the most easily recognized discontinuities across a front is

an increase in cloud coverage.
a change in temperature.
an increase in relative humidity.
3423[C]

One weather phenomenon which will always occur when flying across a front is a change in the

stability of the air mass.
type of precipitation.
wind direction.
3424[C]

Steady precipitation preceding a front is an indication of

cumuliform clouds with little or no turbulence.
stratiform clouds with moderate turbulence.
stratiform clouds with little or no turbulence.
3425[C]

Possible mountain wave turbulence could be anticipated when winds of 40 knots or greater blow

down a mountain valley, and the air is unstable.
parallel to a mountain peak, and the air is stable.
across a mountain ridge, and the air is stable.
3426[C]

Where does wind shear occur

Only at lower altitudes.
Only at higher altitudes.
At all altitudes, in all directions.
3427[B]

When may hazardous wind shear be expected

When stable air crosses a mountain barrier where it tends to flow in layers forming lenticular clouds.
In areas of low-level temperature inversion, frontal zones, and clear air turbulence.
Following frontal passage when stratocumulus clouds form indicating mechanical mixing.
3428[C]

A pilot can expect a wind-shear zone in a temperature inversion whenever the windspeed at 2,000 to 4,000 feet above the surface is at least

10 knots.
15 knots.
25 knots.
3429[A]

One in-flight condition necessary for structural icing to form is

visible moisture.
stratiform clouds.
small temperature/dewpoint spread.
3430[B]

In which environment is aircraft structural ice most likely to have the highest accumulation rate

Freezing drizzle.
Freezing rain.
Cumulus clouds with below freezing temperatures.
3431[C]

Why is frost considered hazardous to flight

Frost slows the airflow over the airfoils, thereby increasing control effectiveness.
Frost changes the basic aerodynamic shape of the airfoils, thereby decreasing lift.
Frost spoils the smooth flow of air over the wings, thereby decreasing lifting capability.
3432[C]

How does frost affect the lifting surfaces of an airplane on takeoff

Frost will change the camber of the wing, increasing lift during takeoff.
Frost may cause the airplane to become airborne with a lower angle of attack at a lower indicated airspeed.
Frost may prevent the airplane from becoming airborne at normal takeoff speed.
3433[A]

The conditions necessary for the formation of cumulonimbus clouds are a lifting action and

unstable, moist air.
either stable or unstable air.
unstable air containing an excess of condensation nuclei.
3434[A]

What feature is normally associated with the cumulus stage of a thunderstorm

Continuous updraft.
Roll cloud.
Frequent lightning.
3435[B]

Which weather phenomenon signals the beginning of the mature stage of a thunderstorm

Maximum growth rate of the clouds.
Precipitation beginning to fall.
The appearance of an anvil top.
3436[B]

What conditions are necessary for the formation of thunderstorms

High humidity, high temperature, and cumulus clouds.
High humidity, lifting force, and unstable conditions.
Lifting force, moist air, and extensive cloud cover.
3437[B]

During the life cycle of a thunderstorm, which stage is characterized predominately by downdrafts

Cumulus.
Dissipating.
Mature.
3438[B]

Thunderstorms reach their greatest intensity during the

cumulus stage.
mature stage.
downdraft stage.
3439[A]

Thunderstorms which generally produce the most intense hazard to aircraft are

squall line thunderstorms.
warm front thunderstorms.
steady-state thunderstorms.
3440[A]

A nonfrontal, narrow band of active thunderstorms that often develop ahead of a cold front is a known as a

squall line.
prefrontal system.
dry line.
3441[B]

If there is thunderstorm activity in the vicinity of an airport at which you plan to land, which hazardous atmospheric phenomenon might be expected on the landing approach

Steady rain.
Wind-shear turbulence.
Precipitation static.
3442[C]

Upon encountering severe turbulence, which flight condition should the pilot attempt to maintain

Constant altitude and airspeed.
Constant angle of attack.
Level flight attitude.
3443[B]

What situation is most conducive to the formation of radiation fog

The movement of cold air over much warmer water.
Warm, moist air over low, flatland areas on clear, calm nights.
Moist, tropical air moving over cold, offshore water.
3444[C]

If the temperature/dewpoint spread is small and decreasing, and the temperature is 62 °F, what type weather is most likely to develop

Thunderstorms.
Freezing precipitation.
Fog or low clouds.
3445[C]

In which situation is advection fog most likely to form

A light breeze blowing colder air out to sea.
A warm, moist air mass on the windward side of mountains.
An air mass moving inland from the coast in winter.
3446[C]

What types of fog depend upon wind in order to exist

Radiation fog and ice fog.
Steam fog and ground fog.
Advection fog and upslope fog.
3447[C]

Low-level turbulence can occur and icing can become hazardous in which type of fog

Rain-induced fog.
Upslope fog.
Steam fog.
3448[A]

The development of thermals depends upon

solar heating.
temperature inversions.
a counterclockwise circulation of air.
3449[B]

Which is considered to be the most hazardous condition when soaring in the vicinity of thunderstorms

Static electricity.
Wind shear and turbulence.
Lightning.
3450[C]

Convective circulation patterns associated with sea breezes are caused by

warm, dense air moving inland from over the water.
water absorbing and radiating heat faster than the land.
cool, dense air moving inland from over the water.
3452[B]

Which weather phenomenon is always associated with a thunderstorm

Hail.
Lightning.
Heavy rain.
3453[A]

Individual forecasts for specific routes of flight can be obtained from which weather source

Transcribed Weather Broadcasts (TWEB's).
Area Forecasts.
Terminal Forecasts.
3454[A]

Transcribed Weather Broadcasts (TWEB's) may be monitored by tuning the appropriate radio receiver to certain

VOR and NDB frequencies.
airport advisory frequencies.
ATIS frequencies.
3455[C]

When telephoning a weather briefing facility for preflight weather information, pilots should state

fuel on board.
true airspeed.
the aircraft identification or the pilot's name.
3456[A]

To get a complete weather briefing for the planned flight, the pilot should request

a standard briefing.
a general briefing.
an abbreviated briefing.
3457[B]

Which type weather briefing should a pilot request, when departing within the hour, if no preliminary weather information has been received

Outlook briefing.
Standard briefing.
Abbreviated briefing.
3458[A]

Which type of weather briefing should a pilot request to supplement mass disseminated data

An abbreviated briefing.
An outlook briefing.
A supplemental briefing.
3459[B]

To update a previous weather briefing, a pilot should request

a standard briefing.
an abbreviated briefing.
an outlook briefing.
3460[A]

A weather briefing that is provided when the information requested is 6 or more hours in advance of the proposed departure time is

an outlook briefing.
a prognostic briefing.
a forecast briefing.
3461[A]

When requesting weather information for the following morning, a pilot should request

an outlook briefing.
a standard briefing.
an abbreviated briefing.
3462[A]

(Refer to figure 12.) Which of the reporting stations have VFR weather

KINK, KBOI, and KLAX.
KINK, KBOI, and KJFK.
All.
3463[C]

For aviation purposes, ceiling is defined as the height above the Earth's surface of the

lowest layer of clouds reported as scattered, broken, or thin.
lowest reported obscuration and the highest layer of clouds reported as overcast.
lowest broken or overcast layer or vertical visibility into an obscuration.
3464[A]

(Refer to figure 12.) The wind direction and velocity at KJFK is from

180° true at 4 knots.
040° true at 18 knots.
180° magnetic at 4 knots.
3465[C]

(Refer to figure 12.) What are the wind conditions at Wink, Texas (KINK)

Calm.
111° at 2 knots, gusts 18 knots.
110° at 12 knots, gusts 18 knots.
3466[C]

(Refer to figure 12.) The remarks section for KMDW has RAB35 listed. This entry means

blowing mist has reduced the visibility to 1-1/2 SM.
the barometer has risen .35" Hg.
rain began at 1835Z.
3467[A]

(Refer to figure 12.) What are the current conditions depicted for Chicago Midway Airport (KMDW)

Sky 700 feet overcast, visibility 1-1/2SM, rain.
Sky 700 feet overcast, visibility 11, occasionally 2SM, with rain.
Sky 7000 feet overcast, visibility 1-1/2SM, heavy rain.
3472[A]

(Refer to figure 14.) The base and tops of the overcast layer reported by a pilot are

7,200 feet MSL and 8,900 feet MSL.
1,800 feet MSL and 5,500 feet MSL.
5,500 feet AGL and 7,200 feet MSL.
3473[B]

(Refer to figure 14.) The wind and temperature at 12,000 feet MSL as reported by a pilot are

009° at 121 MPH and 90 °F.
090° at 21 knots and -9 °C.
090° at 21 knots and -9 °F.
3474[B]

(Refer to figure 14.) If the terrain elevation is 1,295 feet MSL, what is the height above ground level of the base of the ceiling

1,295 feet AGL.
505 feet AGL.
6,586 feet AGL.
3475[B]

(Refer to figure 14.) The intensity of the turbulence reported at a specific altitude is

moderate at 5,500 feet and at 7,200 feet.
moderate from 5,500 feet to 7,200 feet.
light to moderate from 7,200 feet to 8,900 feet.
3476[C]

(Refer to figure 14.) The intensity and type of icing reported by a pilot is

light to moderate.
moderate rime.
light to moderate clear.
3478[B]

From which primary source should information be obtained regarding expected weather at the estimated time of arrival if your destination has no Terminal Forecast

Weather Depiction Chart.
Area Forecast.
Low-Level Prognostic Chart.
3479[C]

(Refer to figure 15.) What is the valid period for the TAF for KMEM

1200Z to 1800Z.
1200Z to 1200Z.
1800Z to 1800Z.
3480[C]

(Refer to figure 15.) In the TAF for KMEM, what does "SHRA" stand for

A significant change in precipitation is possible.
A shift in wind direction is expected.
Rain showers.
3481[A]

(Refer to figure 15.) Between 1000Z and 1200Z the visibility at KMEM is forecast to be

3 statute miles.
1/2 statute mile.
6 statute miles.
3482[C]

(Refer to figure 15.) What is the forecast wind for KMEM from 1600Z until the end of the forecast

No significant wind.
020° at 8 knots.
Variable in direction at 4 knots.
3483[B]

(Refer to figure 15.) In the TAF from KOKC, the "FM (FROM) Group" is

forecast for the hours from 1600Z to 2200Z with the wind from 160° at 10 knots, becoming 210° at 15 knots.
forecast for the hours from 1600Z to 2200Z with the wind from 160° at 10 knots.
forecast for the hours from 1600Z to 2200Z with the wind from 160° at 10 knots, becoming 220° at 13 knots with gusts to 20 knots.
3484[A]

(Refer to figure 15.) In the TAF from KOKC, the clear sky becomes

overcast at 2,000 feet during the forecast period between 2200Z and 2400Z.
overcast at 200 feet with the probability of becoming overcast at 400 feet during the forecast period between 2200Z and 2400Z.
overcast at 200 feet with a 40% probability of becoming overcast at 600 feet during the forecast period between 2200Z and 2400Z.
3485[C]

(Refer to figure 15.) During the time period from 0600Z to 0800Z, what significant weather is forecast for KOKC

Visibility - possibly 6 statute miles with scattered clouds at 4,000 feet.
Wind - 210° at 15 knots.
No significant weather is forecast for this time period.
3486[B]

(Refer to figure 15.) The only cloud type forecast in TAF reports is

Scattered cumulus.
Cumulonimbus.
Nimbostratus.
3487[A]

To best determine general forecast weather conditions over several states, the pilot should refer to

Area Forecasts.
Weather Depiction Charts.
Satellite Maps.
3488[A]

(Refer to figure 16.) What is the forecast ceiling and visibility for Tennessee from 2300Z through 0500Z

3,000 feet or greater, and 5 miles or greater.
500 feet to less than 1,000 feet, and 1 mile to less than 3 miles.
1,000 to 3,000 feet, and 3 to 5 miles.
3489[C]

To determine the freezing level and areas of probable icing aloft, the pilot should refer to the

Weather Depiction Chart.
Radar Summary Chart.
Area Forecast.
3490[B]

The section of the Area Forecast entitled "SIG CLDS AND WX'' contains a summary of

weather advisories still in effect at the time of issue.
cloudiness and weather significant to flight operations broken down by states or other geographical areas.
forecast sky cover, cloud tops, visibility, and obstructions to vision along specific routes.
3491[C]

(Refer to figure 16.) What hazards are forecast in the Area Forecast for TN, AL, and the coastal waters

Thunderstorms with severe or greater turbulence, severe icing, and low-level wind shear.
Moderate rime icing above the freezing level to 10,000 feet.
Moderate turbulence from 25,000 to 38,000 feet due to the jetstream.
3492[A]

(Refer to figure 16.) What type obstructions to vision, if any, are forecast for the entire area from 2300Z until 0500Z the next day

None of any significance, VFR is forecast.
Visibility 3 to 5 miles in fog.
Visibility below 3 miles in fog over south-central Texas.
3493[B]

(Refer to figure 16.) What sky condition and type obstructions to vision are forecast for all the area except TN from 1040Z until 2300Z

Generally ceilings 3,000 to 8,000 feet to clear with visibility sometimes below 3 miles in fog.
8,000 feet scattered to clear except visibility below 3 miles in fog until 1500Z over south-central Texas.
Ceilings 3,000 to 5,000 feet broken, visibility 3 to 5 miles in fog.
3494[A]

To obtain a continuous transcribed weather briefing, including winds aloft and route forecasts for a cross-country flight, a pilot should monitor a

Transcribed Weather Broadcast (TWEB) on an ADF radio receiver.
regularly scheduled weather broadcast on a VOR frequency.
VHF radio receiver tuned to an Automatic Terminal Information Service (ATIS) frequency.
3495[C]

What is indicated when a current CONVECTIVE SIGMET forecasts thunderstorms

Moderate thunderstorms covering 30 percent of the area.
Moderate or severe turbulence.
Thunderstorms obscured by massive cloud layers.
3496[B]

What information is contained in a CONVECTIVE SIGMET

Surface winds greater than 40 knots or thunderstorms equal to or greater than video integrator processor (VIP) level 4.
Tornadoes, embedded thunderstorms, and hail 3/4 inch or greater in diameter.
Severe icing, severe turbulence, or widespread dust storms lowering visibility to less than 3 miles.
3497[B]

SIGMET's are issued as a warning of weather conditions hazardous to which aircraft

Large aircraft only.
All aircraft.
Small aircraft only.
3498[C]

Which in-flight advisory would contain information on severe icing not associated with thunderstorms

Convective SIGMET.
AIRMET.
SIGMET.
3499[A]

AIRMET's are issued as a warning of weather conditions particularly hazardous to which aircraft

Small single-engine aircraft.
All aircraft.
Large multiengine aircraft.
3500[A]

(Refer to figure 17.) What wind is forecast for STL at 6,000 feet?

230° true at 25 knots.
210° magnetic at 13 knots.
232° true at 5 knots.
3501[C]

(Refer to figure 17.) What wind is forecast for STL at 18,000 feet

235° magnetic at 06, peak gusts to 16 knots.
235° true at 06 knots.
230° true at 56 knots.
3502[A]

(Refer to figure 17.) Determine the wind and temperature aloft forecast for DEN at 30,000 feet.

230° true at 53 knots, temperature -47 °C.
235° true at 34 knots, temperature -7 °C.
023° magnetic at 53 knots, temperature 47 °C.
3503[A]

(Refer to figure 17.) Determine the wind and temperature aloft forecast for 3,000 feet at MKC

050° true at 7 knots, temperature missing.
360° magnetic at 5 knots, temperature -7 °C.
360° true at 50 knots, temperature +7 °C.
3504[B]

(Refer to figure 17.) What wind is forecast for STL at 34,000 feet

073° true at 6 knots.
230° true at 106 knots.
007° magnetic at 30 knots.
3505[A]

What values are used for Winds Aloft Forecasts

True direction and knots.
Magnetic direction and miles per hour.
Magnetic direction and knots.
3506[C]

When the term "light and variable'' is used in reference to a Winds Aloft Forecast, the coded group and windspeed is

0000 and less than 7 knots.
9999 and less than 10 knots.
9900 and less than 5 knots.
3507[C]

(Refer to figure 18.) What is the status of thefront that extends from New Mexico to Indiana

Retreating.
Occluded.
Stationary.
3508[C]

(Refer to figure 18.) The IFR weather in eastern Texas is due to

dust devils.
intermittent rain.
fog.
3509[C]

(Refer to figure 18.) Of what value is the Weather Depiction Chart to the pilot

For a forecast of cloud coverage, visibilities, and frontal activity.
For determining frontal trends and air mass characteristics.
For determining general weather conditions on which to base flight planning.
3510[C]

(Refer to figure 18.) The marginal weather in southeast New Mexico is due to

reported thunderstorms.
600-foot overcast ceilings.
low visibility.
3511[C]

(Refer to figure 18.) What weather phenomenon is causing IFR conditions along the coast of Oregon and California

Squall line activity.
Heavy rain showers.
Low ceilings.
3512[B]

(Refer to figure 18.) According to the Weather Depiction Chart, the weather for a flight from central Arkansas to southeast Alabama is

broken clouds at 2,500 feet.
broken to scattered clouds at 25,000 feet.
visibility from 3 to 5 miles.
3513[A]

Radar weather reports are of special interest to pilots because they indicate

location of precipitation along with type, intensity, and trend.
large areas of low ceilings and fog.
location of broken to overcast clouds.
3514[C]

What information is provided by the Radar Summary Chart that is not shown on other weather charts

Types of clouds between reporting stations.
Ceilings and precipitation between reporting stations.
Lines and cells of hazardous thunderstorms.
3515[A]

(Refer to figure 19, area A.) What is the direction and speed of movement of the radar return

East at 15 knots.
020° at 20 knots.
Northeast at 22 knots.
3516[A]

(Refer to figure 19, area C.) What type of weather is occurring in the radar return

Rain showers increasing in intensity.
Heavy rain showers.
Continuous rain.
3517[C]

(Refer to figure 19, area D.) What is the direction and speed of movement of the radar return

West at 30 knots.
Southeast at 30 knots.
Northeast at 20 knots.
3518[B]

(Refer to figure 19, area D.) The top of the precipitation is

2,000 feet.
30,000 feet.
20,000 feet.
3519[B]

(Refer to figure 19, area B.) What does the dashed line enclose

Areas of hail 1/4 inch in diameter.
Severe weather watch area.
Areas of heavy rain.
3520[B]

(Refer to figure 20.) How are Significant Weather Prognostic Charts best used by a pilot

For analyzing current frontal activity and cloud coverage.
For determining areas to avoid (freezing levels and turbulence).
For overall planning at all altitudes.
3521[C]

(Refer to figure 20.) Interpret the weather symbol depicted in southern California on the 12-hour Significant Weather Prognostic Chart.

Base of clear air turbulence, 18,000 feet.
Thunderstorm tops at 18,000 feet.
Moderate turbulence, surface to 18,000 feet.
3522[B]

(Refer to figure 20.) What weather is forecast for the Gulf Coast area just ahead of the cold front during the first 12 hours

IFR with moderate or greater turbulence over the coastal areas.
Ceiling 1,000 to 3,000 feet and/or visibility 3 to 5 miles with intermittent thundershowers and rain showers.
Rain and thunderstorms moving northeastward ahead of the front.
3523[A]

(Refer to figure 20.) The low pressure associated with the cold front in the western states is forecast to move

east at 30 knots.
northeast at 12 knots.
southeast at 30 knots.
3524[A]

(Refer to figure 20.) At what altitude is the freezing level over northeastern Oklahoma on the 24-hour Significant Weather Prognostic Chart

8,000 feet.
4,000 feet.
10,000 feet.
3526[C]

What should pilots state initially when telephoning a weather briefing facility for preflight weather information

Tell the number of occupants on board.
State their total flight time.
Identify themselves as pilots.
3528[B]

When telephoning a weather briefing facility for preflight weather information, pilots should state

the full name and address of the formation commander.
whether they intend to fly VFR only.
that they possess a current pilot certificate.
3529[A]

(Refer to figure 21.) En route to First Flight Airport (area 5), your flight passes over Hampton Roads Airport (area 2) at 1456 and then over Chesapeake Municipal at 1501. At what time should your flight arrive at First Flight

1526.
1521.
1516.
3530[C]

(Refer to figure 21, area 3.) Determine the approximate latitude and longitude of Currituck County Airport.

47°24'N - 75°58'W.
36°48'N - 76°01'W.
36°24'N - 76°01'W.
3531[B]

(Refer to figure 21.) Determine the magnetic course from First Flight Airport (area 5) to Hampton Roads Airport (area 2).

321°.
330°.
312°.
3532[B]

(Refer to figure 21.) What is your approximate position on low altitude airway Victor 1, southwest of Norfolk (area 1), if the VOR receiver indicates you are on the 340° radial of Elizabeth City VOR (area 3)

23 nautical miles from Norfolk VORTAC.
18 nautical miles from Norfolk VORTAC.
15 nautical miles from Norfolk VORTAC.
3533[C]

(Refer to figure 21, area 3; and figure 29.) The VOR is tuned to Elizabeth City VOR, and the aircraft is positioned over Shawboro. Which VOR indication is correct

6.
5.
8.
3534[C]

(Refer to figure 22.) What is the estimated time en route from Mercer County Regional Airport (area 3) to Minot International (area 1)? The wind is from 330° at 25 knots and the true airspeed is 100 knots. Add 3-1/2 minutes for departure and climb-out.

44 minutes.
52 minutes.
48 minutes.
3535[C]

(Refer to figure 22, area 2.) Which airport is located at approximately 47°39'30"N latitude and 100°53'00"W longitude

Johnson.
Linrud.
Crooked Lake.
3536[B]

(Refer to figure 22, area 3.) Which airport is located at approximately 47°21'N latitude and 101°01'W longitude

Evenson.
Washburn.
Underwood.
3537[A]

(Refer to figure 22.) An airship crosses over Minot VORTAC (area 1) at 1056 and over the creek 8 nautical miles south-southeast on Victor 15 at 1108. What should be the approximate position on Victor 15 at 1211

Crossing the road east of Underwood.
Over Lake Nettie National Wildlife Refuge.
Over the powerlines east of Washburn Airport.
3538[C]

(Refer to figure 22.) Determine the magnetic heading for a flight from Mercer County Regional Airport (area 3) to Minot International (area 1). The wind is from 330° at 25 knots, the true airspeed is 100 knots, and the magnetic variation is 10° east.

012°.
002°.
352°.
3539[C]

(Refer to figure 22.) What course should be selected on the omnibearing selector (OBS) to make a direct flight from Mercer County Regional Airport (area 3) to the Minot VORTAC (area 1) with a TO indication

177°.
357°.
001°.
3540[C]

(Refer to figure 23.) What is the estimated time en route from Wall Airport (area 1) to St. Maries Airport (area 4)? The wind is from 215° at 25 knots, and the true airspeed is 125 knots.

30 minutes.
38 minutes.
34 minutes.
3541[C]

(Refer to figure 23.) Determine the estimated time en route for a flight from Priest River Airport (area 1) to Shoshone County Airport (area 3). The wind is from 030 at 12 knots and the true airspeed is 95 knots. Add 2 minutes for climb-out.

27 minutes.
23 minutes.
31 minutes.
3542[C]

(Refer to figure 23.) What is the estimated time en route for a flight from St. Maries Airport (area 4) to Priest River Airport (area 1)? The wind is from 300° at 14 knots and the true airspeed is 90 knots. Add 3 minutes for climb-out.

38 minutes.
48 minutes.
43 minutes.
3543[B]

(Refer to figure 23, area 3.) Determine the approximate latitude and longitude of Shoshone County Airport.

47°02'N - 116°11'W.
47°32'N - 116°11'W.
47°32'N - 116°41'W.
3544[C]

(Refer to figure 23, area 2.) If a balloon is launched at Ranch Aero (Pvt) Airport with a reported wind from 220° at 5 knots, what should be its approximate position after 2 hours of flight

3-1/2 miles southwest of Rathdrum.
Crossing the railroad southwest of Granite Airport.
Near Hackney (Pvt) Airport.
3546[A]

(Refer to figure 23.) What is the magnetic heading for a flight from Priest River Airport (area 1) to Shoshone County Airport (area 3)? The wind is from 030° at 12 knots, and the true airspeed is 95 knots.

117°.
143°.
131°.
3547[C]

(Refer to figure 23.) Determine the magnetic heading for a flight from St. Maries Airport (area 4) to Priest River Airport (area 1). The wind is from 300° at 14 knots, and the true airspeed is 90 knots.

328°.
339°.
320°.
3548[A]

(Refer to figure 24.) What is the estimated time en route for a flight from Allendale County Airport (area 1) to Claxton-Evans County Airport (area 2)? The wind is from 090° at 16 knots and the true airspeed is 90 knots. Add 2 minutes for climb-out.

37 minutes.
41 minutes.
33 minutes.
3549[C]

(Refer to figure 24.) What is the estimated time en route for a flight from Claxton-Evans County Airport (area 2) to Hampton Varnville Airport (area 1)? The wind is from 290° at 18 knots and the true airspeed is 85 knots. Add 2 minutes for climb-out.

35 minutes.
44 minutes.
39 minutes.
3550[A]

(Refer to figure 24.) Determine the compass heading for a flight from Allendale County Airport (area 1) to Claxton-Evans County Airport (area 2). The wind is from 090° at 16 knots, and the true airspeed is 90 knots.

211°.
229°.
205°.
3552[A]

(Refer to figure 24.) What is the approximate position of the aircraft if the VOR receivers indicate the 310° radial of Savannah VORTAC (area 3) and the 190° radial of Allendale VOR (area 1)

Town of Guyton.
Town of Springfield.
3 miles east of Marlow.
3554[B]

(Refer to figure 24.) While en route on Victor 185, a flight crosses the 248° radial of Allendale VOR at 0951 and then crosses the 216° radial of Allendale VOR at 1000. What is the estimated time of arrival at Savannah VORTAC

1023.
1036.
1028.
3555[C]

(Refer to figure 25.) Estimate the time en route from Majors Airport (area 1) to Winnsboro Airport (area 2). The wind is from 340° at 12 knots and the true airspeed is 36 knots.

55 minutes.
63 minutes.
59 minutes.
3556[C]

(Refer to figure 25). Determine the magnetic course from Airpark East Airport (area 1) to Winnsboro Airport (area 2). Magnetic variation is 6°30'E.

082°.
091°.
075°.
3558[C]

(Refer to figure 25.) Determine the magnetic heading for a flight from Majors Airport (area 1) to Winnsboro Airport (area 2). The wind is from 340° at 12 knots, the true airspeed is 36 knots, and the magnetic variation is 6°30'E.

101°.
091°.
078°.
3559[B]

(Refer to figure 25.) What is the approximate position of the aircraft if the VOR receivers indicate the 245° radial of Sulphur Springs VORTAC (area 2) and the 130° radial of Blue Ridge VORTAC (area 1)

3 miles southeast of Caddo Mills Airport.
Meadowview Airport.
Caddo Mills Airport.
3560[B]

(Refer to figure 25.) On what course should the VOR receiver (OBS) be set in order to navigate direct from Majors Airport (area 1) to Quitman VORTAC (area 2)

281°.
101°.
108°.
3561[B]

(Refer to figure 25, area 1; and figure 29.) The VOR is tuned to Blue Ridge VORTAC, and the aircraft is positioned over the town of Lone Oak, southeast of Majors Airport. Which VOR indication is correct

4.
7.
1.
3562[B]

(Refer to figure 26.) What is the estimated time en route for a flight from Denton Muni (area 1) to Addison (area 2)? The wind is from 200° at 20 knots, the true airspeed is 110 knots, and the magnetic variation is 7° east.

16 minutes.
13 minutes.
19 minutes.
3563[A]

(Refer to figure 26.) Estimate the time en route from Addison (area 2) to Redbird (area 3). The wind is from 300° at 15 knots, the true airspeed is 120 knots, and the magnetic variation is 7° east.

8 minutes.
14 minutes.
11 minutes.
3564[C]

(Refer to figure 26.) Determine the magnetic heading for a flight from Redbird (area 3) to Fort Worth Meacham (area 4). The wind is from 030° at 10 knots, the true airspeed is 35 knots, and the magnetic variation is 7° east.

266°.
312°.
298°.
3565[B]

(Refer to figure 26.) Determine the magnetic heading for a flight from Fort Worth Meacham (area 4) to Denton Muni (area 1). The wind is from 330° at 25 knots, the true airspeed is 110 knots, and the magnetic variation is 7° east.

017°.
003°.
023°.
3566[B]

(Refer to figure 26, area 5.) The VOR is tuned to the Dallas/Fort Worth VORTAC. The omnibearing selector (OBS) is set on 253°, with a TO indication, and a right course deviation indicator (CDI) deflection. What is the aircraft's position from the VORTAC

North-northeast.
East-northeast.
West-southwest.
3567[B]

(Refer to figure 27, area 2.) What is the approximate latitude and longitude of Cooperstown Airport?
47°25'N - 99°54'W.
47°25'N - 98°06'W.
47°55'N - 98°06'W.
3568[C]

(Refer to figure 27.) Determine the magnetic course from Breckheimer (Pvt) Airport (area 1) to Jamestown Airport (area 4).

021°.
013°.
181°.
3569[A]

(Refer to figure 27, area 5.) A balloon drifts over the town of Eckelson on a magnetic course of 282° at 10 MPH. If wind conditions remain constant, where will the balloon be after 2 hours 30 minutes

Over Buchanan.
Over the tower southwest of Fried.
3 miles south-southwest of Buchanan.
3570[B]

(Refer to figure 27, areas 4 and 3; and figure 29.) The VOR is tuned to Jamestown VOR, and the aircraft is positioned over the town of Wimbledon. Which VOR indication is correct

1.
6.
4.
3571[C]

(Refer to figure 28.) An aircraft departs an airport in the eastern daylight time zone at 0945 EDT for a 2-hour flight to an airport located in the central daylight time zone. The landing should be at what coordinated universal time

1445Z.
1345Z.
1545Z.
3572[A]

(Refer to figure 28.) An aircraft departs an airport in the central standard time zone at 0930 CST for a 2-hour flight to an airport located in the mountain standard time zone. The landing should be at what time

1030 MST.
1130 MST.
0930 MST.
3573[C]

(Refer to figure 28.) An aircraft departs an airport in the central standard time zone at 0845 CST for a 2-hour flight to an airport located in the mountain standard time zone. The landing should be at what coordinated universal time

1345Z.
1445Z.
1645Z.
3574[C]

(Refer to figure 28.) An aircraft departs an airport in the mountain standard time zone at 1615 MST for a 2-hour 15-minute flight to an airport located in the Pacific standard time zone. The estimated time of arrival at the destination airport should be

1830 PST.
1630 PST.
1730 PST.
3575[A]

(Refer to figure 28.) An aircraft departs an airport in the Pacific standard time zone at 1030 PST for a 4-hour flight to an airport located in the central standard time zone. The landing should be at what coordinated universal time

2230Z.
2030Z.
2130Z.
3576[C]

(Refer to figure 28.) An aircraft departs an airport in the mountain standard time zone at 1515 MST for a 2-hour 30-minute flight to an airport located in the Pacific standard time zone. What is the estimated time of arrival at the destination airport

1745 PST.
1845 PST.
1645 PST.
3577[B]

(Refer to figure 29, illustration 1.) The VOR receiver has the indications shown. What is the aircraft's position relative to the station

East.
South.
North.
3578[B]

(Refer to figure 29, illustration 3.) The VOR receiver has the indications shown. What is the aircraft's position relative to the station

East.
Southeast.
West.
3579[A]

(Refer to figure 29, illustration 8.) The VOR receiver has the indications shown. What radial is the aircraft crossing

030°.
300°.
210°.
3580[B]

(Refer to figure 30, illustration 1.) Determine the magnetic bearing TO the station.

180°.
210°.
030°.
3581[B]

(Refer to figure 30, illustration 2.) What magnetic bearing should the pilot use to fly TO the station

145°.
190°.
010°.
3582[A]

(Refer to figure 30, illustration 2.) Determine the approximate heading to intercept the 180° bearing TO the station.

220°.
040°.
160°.
3583[B]

(Refer to figure 30, illustration 3.) What is the magnetic bearing FROM the station

025°.
115°.
295°.
3584[B]

(Refer to figure 30.) Which ADF indication represents the aircraft tracking TO the station with a right crosswind

2.
4.
1.
3585[B]

(Refer to figure 30, illustration 1.) What outbound bearing is the aircraft crossing

180°.
030°.
150°.
3586[A]

(Refer to figure 30, illustration 1.) What is the relative bearing TO the station

240°.
030°.
210°.
3587[A]

(Refer to figure 30, illustration 2.) What is the relative bearing TO the station

235°.
190°.
315°.
3588[C]

(Refer to figure 30, illustration 4.) What is the relative bearing TO the station

020°.
060°.
340°.
3589[A]

(Refer to figure 31, illustration 1.) The relative bearing TO the station is

315°.
045°.
180°.
3590[C]

(Refer to figure 31, illustration 2). The relative bearing TO the station is

180°.
270°.
090°.
3591[A]

(Refer to figure 31, illustration 3.) The relative bearing TO the station is

180°.
270°.
090°.
3592[A]

(Refer to figure 31, illustration 4.) On a magnetic heading of 320°, the magnetic bearing TO the station is

185°.
005°.
225°.
3593[B]

(Refer to figure 31, illustration 5) On a magnetic heading of 035°, the magnetic bearing TO the station is

215°.
035°.
180°.
3594[A]

(Refer to figure 31, illustration 6.) On a magnetic heading of 120°, the magnetic bearing TO the station is

165°.
045°.
270°.
3595[B]

(Refer to figure 31, illustration 6.) If the magnetic bearing TO the station is 240°, the magnetic heading is

105°.
195°.
045°.
3596[C]

(Refer to figure 31, illustration 7.) If the magnetic bearing TO the station is 030°, the magnetic heading is

270°.
060°.
120°.
3597[A]

(Refer to figure 31, illustration 8.) If the magnetic bearing TO the station is 135°, the magnetic heading is

360°.
135°.
270°.
3598[A]

When the course deviation indicator (CDI) needle is centered during an omnireceiver check using a VOR test signal (VOT), the omnibearing selector (OBS) and the TO/FROM indicator should read

0° FROM or 180° TO, regardless of the pilot's position from the VOT.
0° TO or 180° FROM, regardless of the pilot's position from the VOT.
180° FROM, only if the pilot is due north of the VOT.
3599[C]

(Refer to figure 26, area 4.) The floor of Class B airspace overlying Hicks Airport (T67) north-northwest of Fort Worth Meacham Field is

at the surface.
3,200 feet MSL.
4,000 feet MSL.
3600[A]

(Refer to figure 26, area 2.) The floor of Class B airspace at Addison Airport is

3,000 feet MSL.
at the surface.
3,100 feet MSL.
3601[B]

(Refer to figure 21.) What hazards to aircraft may exist in warning areas such as Warning W-50B

Heavy military aircraft traffic in the approach and departure area of the North Atlantic Control Area.
Unusual, often invisible, hazards such as aerial gunnery or guided missiles over international waters.
High volume of pilot training or unusual type of aerial activity.
3602[C]

(Refer to figure 27.) What hazards to aircraft may exist in areas such as Devils Lake East MOA

Parachute jump operations.
Unusual, often invisible, hazards to aircraft such as artillery firing.
High density military training activities.
3603[A]

(Refer to figure 22.) What type military flight operations should a pilot expect along IR 644

IFR training flights above 1,500 feet AGL at speeds in excess of 250 knots.
Instrument training flights below 1,500 feet AGL at speeds in excess of 150 knots.
VFR training flights above 1,500 feet AGL at speeds less than 250 knots.
3604[A]

(Refer to figure 21, area 3.) What is the recommended communications procedure for a landing at Currituck County Airport

Transmit intentions on 122.9 MHz when 10 miles out and give position reports in the traffic pattern.
Contact Elizabeth City FSS for airport advisory service.
Contact New Bern FSS for area traffic information.
3605[B]

(Refer to figure 22, area 2.) The CTAF/MULTICOM frequency for Garrison Municipal is

123.0 MHz.
122.9 MHz.
122.8 MHz.
3606[B]

(Refer to figure 23, area 2; and figure 32.) At Coeur D'Alene , which frequency should be used as a Common Traffic Advisory Frequency (CTAF) to self-announce position and intentions

122.1/108.8 MHz.
122.05 MHz.
122.8 MHz.
3607[C]

(Refer to figure 23, area 2; and figure 32.) At Coeur D'Alene, which frequency should be used as a Common Traffic Advisory Frequency (CTAF) to monitor airport traffic

122.1/108.8 MHz.
122.8 MHz.
122.05 MHz.
3608[B]

(Refer to figure 23, area 2; and figure 32.) What is the correct UNICOM frequency to be used at Coeur D'Alene to request fuel

122.1/108.8 MHz.
122.8 MHz.
119.1 MHz.
3609[B]

(Refer to figure 26, area 3.) If Redbird Tower is not in operation, which frequency should be used as a Common Traffic Advisory Frequency (CTAF) to monitor airport traffic

122.95 MHz.
120.3 MHz.
126.35 MHz.
3610[C]

(Refer to figure 27, area 2.) What is the recommended communication procedure when inbound to land at Cooperstown Airport

Circle the airport in a left turn prior to entering traffic.
Contact UNICOM when 10 miles out on 122.8 MHz.
Broadcast intentions when 10 miles out on the CTAF/MULTICOM frequency, 122.9 MHz.
3611[B]

(Refer to figure 27, area 4.) The CTAF/UNICOM frequency at Jamestown Airport is

122.0 MHz.
123.0 MHz.
123.6 MHz.
3612[A]

(Refer to figure 27, area 6.) What is the CTAF/UNICOM frequency at Barnes County Airport

122.8 MHz.
122.0 MHz.
123.6 MHz.
3613[B]

When flying HAWK N666CB, the proper phraseology for initial contact with McAlester AFSS is

"MC ALESTER STATION, HAWK SIX SIX SIX CEE BEE, RECEIVING ARDMORE VORTAC, OVER.''
"MC ALESTER RADIO, HAWK SIX SIX SIX CHARLIE BRAVO, RECEIVING ARDMORE VORTAC, OVER.''
"MC ALESTER FLIGHT SERVICE STATION, HAWK NOVEMBER SIX CHARLIE BRAVO, RECEIVING ARDMORE VORTAC, OVER.''
3614[C]

The correct method of stating 4,500 feet MSL to ATC is

"FORTY-FIVE HUNDRED FEET MSL.''
"FOUR POINT FIVE.''
"FOUR THOUSAND FIVE HUNDRED.''
3615[C]

The correct method of stating 10,500 feet MSL to ATC is

"TEN POINT FIVE.''
"TEN THOUSAND, FIVE HUNDRED FEET.''
"ONE ZERO THOUSAND, FIVE HUNDRED.''
3616[B]

How should contact be established with an En Route Flight Advisory Service (EFAS) station, and what service would be expected

Call flight assistance on 122.5 for advisory service pertaining to severe weather.
Call Flight Watch on 122.0 for information regarding actual weather and thunderstorm activity along proposed route.
Call EFAS on 122.2 for routine weather, current reports on hazardous weather, and altimeter settings.
3617[B]

What service should a pilot normally expect from an En Route Flight Advisory Service (EFAS) station

Preferential routing and radar vectoring to circumnavigate severe weather.
Actual weather information and thunderstorm activity along the route.
Severe weather information, changes to flight plans, and receipt of routine position reports.
3618[A]

(Refer to figure 27, area 3.) When flying over Arrowwood National Wildlife Refuge, a pilot should fly no lower than

2,000 feet AGL.
2,500 feet AGL.
3,000 feet AGL.
3619[B]

(Refer to figure 23, area 2 and legend 1.) For information about the parachute jumping and glider operations at Silverwood Airport, refer to

notes on the border of the chart.
the Airport/Facility Directory.
the Notices to Airmen (NOTAM) publication.
3620[C]

(Refer to figure 23, area 1.) The visibility and cloud clearance requirements to operate VFR during daylight hours over Wall Airport at less than 1,200 feet AGL are

3 miles and 1,000 feet above, 500 feet below, and 2,000 feet horizontally from each cloud.
1 mile and 1,000 feet above, 500 feet below, and 2,000 feet horizontally from each cloud.
1 mile and clear of clouds.
3620[C]

(Refer to figure 23, area 1.) The visibility and cloud clearance requirements to operate over Wall Airport at less than 1,200 feet AGL are

1 mile and 1,000 feet above, 500 feet below, and 2,000 feet horizontally from each cloud.
3 miles and 1,000 feet above, 500 feet below, and 2,000 feet horizontally from each cloud.
3 miles and clear of clouds.
3621[A]

(Refer to figure 27, area 2.) The visibility and cloud clearance requirements to operate VFR during daylight hours over the town of Cooperstown between 1,200 feet AGL and 10,000 feet MSL are

1 mile and 1,000 feet above, 500 feet below, and 2,000 feet horizontally from clouds.
1 mile and clear of clouds.
3 miles and 1,000 feet above, 500 feet below, and 2,000 feet horizontally from clouds.
3621[A]

(Refer to figure 27, area 2.) The visibility and cloud clearance requirements to operate over the town of Cooperstown between 1,200 feet AGL and 10,000 feet MSL are

1 mile and 1,000 feet above, 500 feet below, and 2,000 feet horizontally from clouds.
3 miles and 1,000 feet above, 500 feet below, and 2,000 feet horizontally from clouds.
1 mile and clear of clouds.
3622[C]

(Refer to figure 27, area 1.) Identify the airspace over Lowe Airport that exists from the surface to 14,500 feet MSL.

A. Class G airspace - surface to 3,500 feet MSL; Class E airspace - 3,500 feet MSL to 14,500 feet MSL.
B. Class G airspace - surface to 3,500 feet MSL; Class E airspace - 3,500 feet MSL to 10,000 feet MSL; Class G airspace - 10,000 feet MSL to 14,500 feet MSL.
C. Class G airspace - surface to 14,500 feet MSL.
3623[A]

(Refer to figure 27, area 6.) The airspace overlying and within 5 miles of Barnes County Airport is

Class G airspace from the surface to 700 feet AGL.
Class D airspace from the surface to the floor of the overlying Class E airspace.
Class E airspace from the surface to 1,200 feet MSL.
3624[C]

(Refer to figure 26, area 7.) The airspace overlying McKinney Muni is uncontrolled from the surface to

4,000 feet AGL.
1,700 feet MSL.
700 feet AGL.
3625[B]

(Refer to figure 26, area 4.) The airspace directly overlying Fort Worth Meacham is

Class B airspace to 10,000 feet MSL.
Class D airspace to 3,200 feet MSL.
Class C airspace to 5,000 feet MSL.
3626[C]

(Refer to figure 24, area 3.) What is the floor of the Savannah Class C airspace at the shelf area (outer circle)

1,700 feet MSL.
1,300 feet AGL.
1,300 feet MSL.
3627[A]

(Refer to figure 21, area 1.) What minimum radio equipment is required to land and take off at Norfolk International

Mode C transponder and two-way radio.
Mode C transponder and omnireceiver.
Mode C transponder, omnireceiver, and DME.
3628[B]

(Refer to figure 26.) At which airports is fixed-wing Special VFR not authorized

Fort Worth Meacham and Fort Worth Spinks.
Dallas-Fort Worth International and Dallas Love Field.
Addison and Redbird.
3629[B]

(Refer to figure 23, area 3.) The vertical limits of that portion of Class E airspace designated as a Federal Airway over Magee Airport are

1,200 feet AGL to 10,000 feet MSL.
7,500 feet MSL to 17,999 feet MSL.
7,500 feet MSL to 12,500 feet MSL.
3630[C]

(Refer to figure 22.) On what frequency can a pilot receive Hazardous Inflight Weather Advisory Service (HIWAS) in the vicinity of area 1

118.0 MHz.
122.0 MHz.
117.1 MHz.
3631[A]

(Refer to figure 21, area 5.) The CAUTION box denotes what hazard to aircraft

Cable extending from radar-outfitted balloons.
Tall bridge over the inlet to the body of water.
Guy wires extending from radio or TV towers.
3632[A]

(Refer to figure 21, area 2.) The flag symbol at Lake Drummond represents a

visual checkpoint used to identify position for initial callup to Norfolk Approach Control.
compulsory reporting point for Norfolk Class C airspace.
compulsory reporting point for Hampton Roads Airport.
3633[B]

(Refer to figure 21, area 2.) The elevation of the Chesapeake Municipal Airport is

36 feet.
20 feet.
360 feet.
3634[C]

(Refer to figure 22.) The terrain elevation of the light tan area between Minot (area 1) and Audubon Lake (area 2) varies from

sea level to 2,000 feet MSL.
2,000 feet to 2,700 feet MSL.
2,000 feet to 2,500 feet MSL.
3635[A]

(Refer to figure 22.) Which public use airports depicted are indicated as having fuel

Minot and Mercer County Regional Airport.
Minot and Garrison.
Mercer County Regional Airport and Garrison.
3636[C]

(Refer to figure 24.) The flag symbols at Statesboro Airport, Claxton-Evans County Airport, and Ridgeland Airport are

outer boundaries of Savannah Class C airspace.
airports with special traffic patterns.
visual checkpoints to identify position for initial callup prior to entering Savannah Class C airspace.
3638[B]

(Refer to figure 24, area 3.) The top of the lighted stack approximately 12 nautical miles from the Savannah VORTAC on the 350° radial is

305 feet AGL.
400 feet AGL.
430 feet AGL.
3639[A]

(Refer to figure 25, area 1.) What minimum altitude is necessary to vertically clear the obstacle on the northeast side of Airpark East Airport by 500 feet

1,273 feet MSL.
1,010 feet MSL.
1,283 feet MSL.
3640[A]

(Refer to figure 25, area 2.) What minimum altitude is necessary to vertically clear the obstacle on the southeast side of Winnsboro Airport by 500 feet

1,403 feet MSL.
823 feet MSL.
1,013 feet MSL.
3641[C]

(Refer to figure 26, area 2.) The control tower frequency for Addison Airport is

133.4 MHz.
122.95 MHz.
126.0 MHz.
3642[A]

(Refer to figure 26, area 8.) What minimum altitude is required to fly over the Cedar Hill TV towers in the congested area south of NAS Dallas

3,349 feet MSL.
2,849 feet MSL.
2,533 feet MSL.
3651[A]

What action can a pilot take to aid in cooling an engine that is overheating during a climb

Reduce rate of climb and increase airspeed.
Reduce climb speed and increase RPM.
Increase climb speed and increase RPM.
3652[A]

What is one procedure to aid in cooling an engine that is overheating

Enrichen the fuel mixture.
Increase the RPM.
Reduce the airspeed.
3653[A]

How is engine operation controlled on an engine equipped with a constant-speed propeller

The throttle controls power output as registered on the manifold pressure gauge and the propeller control regulates engine RPM.
The throttle controls engine RPM as registered on the tachometer and the mixture control regulates the power output.
The throttle controls power output as registered on the manifold pressure gauge and the propeller control regulates a constant blade angle.
3654[A]

What is an advantage of a constant-speed propeller

Permits the pilot to select the blade angle for the most efficient performance.
Provides a smoother operation with stable RPM and eliminates vibrations.
Permits the pilot to select and maintain a desired cruising speed.
3655[A]

A precaution for the operation of an engine equipped with a constant-speed propeller is to

avoid high manifold pressure settings with low RPM.
always use a rich mixture with high RPM settings.
avoid high RPM settings with high manifold pressure.
3656[C]

What should be the first action after starting an aircraft engine

Place the magneto or ignition switch momentarily in the OFF position to check for proper grounding.
Test each brake and the parking brake.
Adjust for proper RPM and check for desired indications on the engine gauges.
3657[B]

Should it become necessary to handprop an airplane engine, it is extremely important that a competent pilot

be in the cockpit and call out all commands.
be at the controls in the cockpit.
call "contact'' before touching the propeller.
3661[C]

Which items are included in the empty weight of an aircraft

Full fuel tanks and engine oil to capacity.
Only the airframe, powerplant, and optional equipment.
Unusable fuel and undrainable oil.
3662[A]

An aircraft is loaded 110 pounds over maximum certificated gross weight. If fuel (gasoline) is drained to bring the aircraft weight within limits, how much fuel should be drained

18.4 gallons.
15.7 gallons.
16.2 gallons.
3663[B]

If an aircraft is loaded 90 pounds over maximum certificated gross weight and fuel (gasoline) is drained to bring the aircraft weight within limits, how much fuel should be drained

12 gallons.
15 gallons.
10 gallons.
3664[B]

GIVEN: WEIGHT ARM MOMENT (LB) (IN) (LB-IN)Empty weight 1,495.0101.4151,593.0Pilot and passengers 380.064.0Fuel (30 gal usable no reserve) 96.0The CG is located how far aft of datum

CG 92.44.
CG 94.01.
CG 119.8.
3665[B]

(Refer to figures 33 and 34.) Determine if the airplane weight and balance is within limits.Front seat occupants 340 lbRear seat occupants 295 lbFuel (main wing tanks) 44 galBaggage 56 lb

20 pounds overweight, CG aft of aft limits.
20 pounds overweight, CG within limits.
20 pounds overweight, CG forward of forward limits.
3666[B]

(Refer to figures 33 and 34.) What is the maximum amount of baggage that can be carried when the airplane is loaded as follows?Front seat occupants 387 lbRear seat occupants 293 lbFuel 35 gal

220 pounds.
45 pounds.
63 pounds.
3667[C]

(Refer to figures 33 and 34.) Calculate the weight and balance and determine if the CG and the weight of the airplane are within limits.Front seat occupants 350 lbRear seat occupants 325 lbBaggage 27 lbFuel 35 gal

CG 84.1, within limits.
CG 81.7, out of limits forward.
CG 83.4, within limits.
3668[B]

(Refer to figures 33 and 34.) Determine if the airplane weight and balance is within limits.Front seat occupants 415 lbRear seat occupants 110 lbFuel, main tanks 44 galFuel, aux. tanks 19 galBaggage 32 lb

19 pounds overweight, CG out of limits forward.
Weight within limits, CG out of limits.
19 pounds overweight, CG within limits.
3669[B]

(Refer to figure 35.) What is the maximum amount of baggage that may be loaded aboard the airplane for the CG to remain within the moment envelope? WEIGHT (LB) MOM/1000Empty weight 1,350 51.5Pilot and front passenger 250 ---Rear passengers 400 ---Baggage --- ---Fuel, 30 gal --- ---Oil, 8 qt --- -0.2

110 pounds.
105 pounds.
120 pounds.
3670[C]

(Refer to figure 35.) Calculate the moment of the airplane and determine which category is applicable. WEIGHT (LB) MOM/1000Empty weight 1,350 51.5Pilot and front passenger 310 ---Rear passengers 96 ---Fuel, 38 gal --- ---Oil, 8 qt --- -0.2

81.2, normal category.
79.2, utility category.
80.8, utility category.
3671[A]

(Refer to figure 35.) What is the maximum amount of fuel that may be aboard the airplane on takeoff if loaded as follows? WEIGHT (LB) MOM/1000Empty weight 1,350 51.5Pilot and front passenger 340 ---Rear passengers 310 ---Baggage 45 ---Oil, 8 qt --- ---

40 gallons.
32 gallons.
24 gallons.
3672[A]

(Refer to figure 35.) Determine the moment with the following data: WEIGHT (LB) MOM/1000Empty weight 1,350 51.5Pilot and front passenger 340 ---Fuel (std tanks) Capacity ---Oil, 8 qt --- ---

74.9 pound-inches.
77.6 pound-inches.
69.9 pound-inches.
3673[C]

(Refer to figure 35.) Determine the aircraft loaded moment and the aircraft category. WEIGHT (LB) MOM/1000Empty weight 1,350 51.5Pilot and front passenger 380 ---Fuel, 48 gal 288 ---Oil, 8 qt --- ---

80.4, utility category.
78.2, normal category.
79.2, normal category.
3674[A]

(Refer to figures 33 and 34.) Upon landing, the front passenger (180 pounds) departs the airplane. A rear passenger (204 pounds) moves to the front passenger position. What effect does this have on the CG if the airplane weighed 2,690 pounds and the MOM/100 was 2,260 just prior to the passenger transfer

The CG moves forward approximately 3 inches.
The weight changes, but the CG is not affected.
The CG moves forward approximately 0.1 inch.
3675[B]

(Refer to figures 33 and 34.) Which action can adjust the airplane's weight to maximum gross weight and the CG within limits for takeoff? Front seat occupants 425 lbRear seat occupants 300 lbFuel, main tanks 44 gal

Drain 12 gallons of fuel.
Drain 9 gallons of fuel.
Transfer 12 gallons of fuel from the main tanks to the auxiliary tanks.
3676[C]

(Refer to figures 33 and 34.) What effect does a 35-gallon fuel burn (main tanks) have on the weight and balance if the airplane weighed 2,890 pounds and the MOM/100 was 2,452 at takeoff

Weight is reduced to 2,680 pounds and the CG moves forward.
Weight is reduced by 210 pounds and the CG is unaffected.
Weight is reduced by 210 pounds and the CG is aft of limits.
3677[A]

(Refer to figures 33 and 34.) With the airplane loaded as follows, what action can be taken to balance the airplane?Front seat occupants 411 lbRear seat occupants 100 lbMain wing tanks 44 gal

Add a 100-pound weight to the baggage compartment.
Transfer 10 gallons of fuel from the main tanks to the auxiliary tanks.
Fill the auxiliary wing tanks.
3678[B]

(Refer to figure 36.) Approximately what true airspeed should a pilot expect with 65 percent maximum continuous power at 9,500 feet with a temperature of 36 °F below standard

178 MPH.
183 MPH.
181 MPH.
3679[A]

(Refer to figure 36.) What is the expected fuel consumption for a 1,000-nautical mile flight under the following conditions?Pressure altitude 8,000 ftTemperature 22 °CManifold pressure 20.8" HgWind Calm

70.1 gallons.
60.2 gallons.
73.2 gallons.
3680[B]

(Refer to figure 36.) What is the expected fuel consumption for a 500-nautical mile flight under the following conditions?Pressure altitude 4,000 ftTemperature +29 °CManifold pressure 21.3" HgWind Calm

40.1 gallons.
36.1 gallons.
31.4 gallons.
3681[A]

(Refer to figure 36.) What fuel flow should a pilot expect at 11,000 feet on a standard day with 65 percent maximum continuous power

11.2 gallons per hour.
10.6 gallons per hour.
11.8 gallons per hour.
3682[C]

(Refer to figure 36.) Determine the approximate manifold pressure setting with 2,450 RPM to achieve 65 percent maximum continuous power at 6,500 feet with a temperature of 36 °F higher than standard.

20.8" Hg.
19.8" Hg.
21.0" Hg.
3683[B]

(Refer to figure 37.) What is the headwind component for a landing on Runway 18 if the tower reports the wind as 220° at 30 knots

26 knots.
23 knots.
19 knots.
3684[B]

(Refer to figure 37.) Determine the maximum wind velocity for a 45° crosswind if the maximum crosswind component for the airplane is 25 knots.

25 knots.
35 knots.
29 knots.
3685[C]

(Refer to figure 37.) What is the maximum wind velocity for a 30° crosswind if the maximum crosswind component for the airplane is 12 knots

16 knots.
20 knots.
24 knots.
3686[B]

(Refer to figure 37.) With a reported wind of north at 20 knots, which runway (6, 29, or 32) is acceptable for use for an airplane with a 13-knot maximum crosswind component

Runway 29.
Runway 32.
Runway 6.
3687[C]

(Refer to figure 37.) With a reported wind of south at 20 knots, which runway (10, 14, or 24) is appropriate for an airplane with a 13-knot maximum crosswind component

Runway 24.
Runway 10.
Runway 14.
3688[B]

(Refer to figure 37.) What is the crosswind component for a landing on Runway 18 if the tower reports the wind as 220° at 30 knots

30 knots.
19 knots.
23 knots.
3689[A]

(Refer to figure 38.) Determine the total distance required to land.OAT 32 °FPressure altitude 8,000 ftWeight 2,600 lbHeadwind component 20 ktsObstacle 50 ft

1,400 feet.
850 feet.
1,750 feet.
3691[B]

(Refer to figure 38.) Determine the total distance required to land.OAT 90 °FPressure altitude 3,000 ftWeight 2,900 lbHeadwind component 10 ktsObstacle 50 ft

1,450 feet.
1,725 feet.
1,550 feet.
3692[A]

(Refer to figure 38.) Determine the approximate total distance required to land over a 50-foot obstacle.OAT 90 °FPressure altitude 4,000 ftWeight 2,800 lbHeadwind component 10 kts

1,950 feet.
1,525 feet.
1,775 feet.
3693[A]

(Refer to figure 39.) Determine the approximate landing ground roll distance.Pressure altitude Sea levelHeadwind 4 ktsTemperature Std

401 feet.
490 feet.
356 feet.
3694[C]

(Refer to figure 39.) Determine the total distance required to land over a 50-foot obstacle.Pressure altitude 7,500 ftHeadwind 8 ktsTemperature 32°FRunway Hard surface

1,506 feet.
1,004 feet.
1,205 feet.
3695[C]

(Refer to figure 39.) Determine the total distance required to land over a 50-foot obstacle.Pressure altitude 5,000 ftHeadwind 8 ktsTemperature 41 °FRunway Hard surface

837 feet.
1,076 feet.
956 feet.
3698[B]

(Refer to figure 39.) Determine the approximate landing ground roll distance.Pressure altitude 1,250 ftHeadwind 8 ktsTemperature Std

275 feet.
366 feet.
470 feet.
3705[B]

(Refer to figure 41.) Determine the total distance required for takeoff to clear a 50-foot obstacle.OAT StdPressure altitude 4,000 ftTakeoff weight 2,800 lbHeadwind component Calm

2,000 feet.
1,750 feet.
1,500 feet.
3706[C]

(Refer to figure 41.) Determine the total distance required for takeoff to clear a 50-foot obstacle.OAT StdPressure altitude Sea levelTakeoff weight 2,700 lbHeadwind component Calm

1,000 feet.
1,700 feet.
1,400 feet.
3707[A]

(Refer to figure 41.) Determine the approximate ground roll distance required for takeoff.OAT 100 °FPressure altitude 2,000 ftTakeoff weight 2,750 lbHeadwind component Calm

1,150 feet.
1,300 feet.
1,800 feet.
3708[B]

(Refer to figure 41.) Determine the approximate ground roll distance required for takeoff.OAT 90 °FPressure altitude 2,000 ftTakeoff weight 2,500 lbHeadwind component 20 kts

850 feet.
650 feet.
1,000 feet.
3709[C]

FAA advisory circulars (some free, others at cost) are available to all pilots and are obtained by

distribution from the nearest FAA district office.
subscribing to the Federal Register.
ordering those desired from the Government Printing Office.
3710[B]

Prior to starting each maneuver, pilots should

check altitude, airspeed, and heading indications.
visually scan the entire area for collision avoidance.
announce their intentions on the nearest CTAF.
3711[A]

The most important rule to remember in the event of a power failure after becoming airborne is to

immediately establish the proper gliding attitude and airspeed.
determine the wind direction to plan for the forced landing.
quickly check the fuel supply for possible fuel exhaustion.
3712[C]

What is the most effective way to use the eyes during night flight

Concentrate directly on each object for a few seconds.
Look only at far away, dim lights.
Scan slowly to permit offcenter viewing.
3713[A]

The best method to use when looking for other traffic at night is to

look to the side of the object and scan slowly.
look to the side of the object and scan rapidly.
scan the visual field very rapidly.
3714[C]

The most effective method of scanning for other aircraft for collision avoidance during nighttime hours is to use

a series of short, regularly spaced eye movements to search each 30-degree sector.
regularly spaced concentration on the 3-, 9-, and 12-o'clock positions.
peripheral vision by scanning small sectors and utilizing offcenter viewing.
3715[A]

During a night flight, you observe a steady red light and a flashing red light ahead and at the same altitude. What is the general direction of movement of the other aircraft

The other aircraft is crossing to the left.
The other aircraft is crossing to the right.
The other aircraft is approaching head-on.
3716[A]

During a night flight, you observe a steady white light and a flashing red light ahead and at the same altitude. What is the general direction of movement of the other aircraft

The other aircraft is flying away from you.
The other aircraft is crossing to the right.
The other aircraft is crossing to the left.
3717[B]

During a night flight, you observe steady red and green lights ahead and at the same altitude. What is the general direction of movement of the other aircraft

The other aircraft is flying away from you.
The other aircraft is approaching head-on.
The other aircraft is crossing to the left.
3718[A]

Airport taxiway edge lights are identified at night by

blue omnidirectional lights.
white directional lights.
alternate red and green lights.
3719[C]

VFR approaches to land at night should be accomplished

with a steeper descent.
at a higher airspeed.
the same as during daytime.
3759[A]

To use VHF/DF facilities for assistance in locating an aircraft's position, the aircraft must have a

VHF transmitter and receiver.
4096-code transponder.
VOR receiver and DME.
3760[B]

A slightly high glide slope indication from a precision approach path indicator is

four white lights.
three white lights and one red light.
two white lights and two red lights.
3761[B]

A below glide slope indication from a tri-color VASI is a

green light signal.
red light signal.
pink light signal.
3762[A]

An above glide slope indication from a tri-color VASI is

an amber light signal.
a white light signal.
a green light signal.
3763[A]

An on glide slope indication from a tri-color VASI is

a green light signal.
an amber light signal.
a white light signal.
3764[C]

A below glide slope indication from a pulsating approach slope indicator is a

steady white light.
pulsating white light.
pulsating red light.
3765[C]

(Refer to figure 48.) Illustration A indicates that the aircraft is

above the glide slope.
below the glide slope.
on the glide slope.
3766[C]

(Refer to figure 48.) VASI lights as shown by illustration C indicate that the airplane is

below the glide slope.
off course to the left.
above the glide slope.
3767[B]

(Refer to figure 48.) While on final approach to a runway equipped with a standard 2-bar VASI, the lights appear as shown by illustration D. This means that the aircraft is

above the glide slope.
below the glide slope.
on the glide slope.
3768[B]

To set the high intensity runway lights on medium intensity, the pilot should click the microphone seven times, then click it

three times.
five times.
one time.
3769[C]

An airport's rotating beacon operated during daylight hours indicates

the Air Traffic Control tower is not in operation.
there are obstructions on the airport.
that weather at the airport located in Class D airspace is below basic VFR weather minimums.
3771[B]

A military air station can be identified by a rotating beacon that emits

white and green alternating flashes.
two quick, white flashes between green flashes.
green, yellow, and white flashes.
3772[A]

How can a military airport be identified at night

Dual peaked (two quick) white flashes between green flashes.
Alternate white and green light flashes.
White flashing lights with steady green at the same location.
3773[A]

(Refer to figure 49.) That portion of the runway identified by the letter A may be used for

taxiing and takeoff.
landing.
taxiing and landing.
3774[C]

(Refer to figure 49.) According to the airport diagram, which statement is true

Runway 30 is equipped at position E with emergency arresting gear to provide a means of stopping military aircraft.
The takeoff and landing portion of Runway 12 begins at position B.
Takeoffs may be started at position A on Runway 12, and the landing portion of this runway begins at position B.
3775[B]

(Refer to figure 49.) What is the difference between area A and area E on the airport depicted

"A'' may be used for all operations except heavy aircraft landings; "E'' may be used only as an overrun.
"A'' may be used for taxi and takeoff; "E'' may be used only as an overrun.
"A'' may be used only for taxiing; "E'' may be used for all operations except landings.
3776[B]

(Refer to figure 49.) Area C on the airport depicted is classified as a

stabilized area.
closed runway.
multiple heliport.
3777[C]

(Refer to figure 50.) The arrows that appear on the end of the north/south runway indicate that the area

may be used only for taxiing.
is usable for taxiing, takeoff, and landing.
cannot be used for landing, but may be used for taxiing and takeoff.
3778[A]

The numbers 9 and 27 on a runway indicate that the runway is oriented approximately

090° and 270° magnetic.
090° and 270° true.
009° and 027° true.
3779[A]

The vertical limit of Class C airspace above the primary airport is normally

4,000 feet AGL.
1,200 feet AGL.
3,000 feet AGL.
3780[C]

The normal radius of the outer area of Class C airspace is

5 nautical miles.
15 nautical miles.
20 nautical miles.
3781[B]

All operations within Class C airspace must be in

compliance with ATC clearances and instructions.
an aircraft equipped with a 4096-code transponder with Mode C encoding capability.
accordance with instrument flight rules.
3782[B]

Under what condition may an aircraft operate from a satellite airport within Class C airspace

The pilot must monitor ATC until clear of the Class C airspace.
The pilot must contact ATC as soon as practicable after takeoff.
The pilot must file a flight plan prior to departure.
3785[A]

What action should a pilot take when operating under VFR in a Military Operations Area (MOA)

Exercise extreme caution when military activity is being conducted.
Operate only on the airways that transverse the MOA.
Obtain a clearance from the controlling agency prior to entering the MOA.
3786[A]

Responsibility for collision avoidance in an alert area rests with

all pilots.
Air Traffic Control.
the controlling agency.
3787[C]

The lateral dimensions of Class D airspace are based on

the number of airports that lie within the Class D airspace.
5 statute miles from the geographical center of the primary airport.
the instrument procedures for which the controlled airspace is established.
3788[B]

A non-tower satellite airport, within the same Class D airspace as that designated for the primary airport, requires radio communications be established and maintained with the

associated Flight Service Station.
primary airport's control tower.
satellite airport's UNICOM.
3789[A]

Prior to entering an Airport Advisory Area, a pilot should

contact the local FSS for airport and traffic advisories.
contact approach control for vectors to the traffic pattern.
monitor ATIS for weather and traffic advisories.
3791[A]

Automatic Terminal Information Service (ATIS) is the continuous broadcast of recorded information concerning

noncontrol information in selected high-activity terminal areas.
nonessential information to reduce frequency congestion.
pilots of radar-identified aircraft whose aircraft is in dangerous proximity to terrain or to an obstruction.
3792[B]

An ATC radar facility issues the following advisory to a pilot flying on a heading of 090°:''TRAFFIC 3 O'CLOCK, 2 MILES, WESTBOUND...''Where should the pilot look for this traffic

West.
South.
East.
3793[B]

An ATC radar facility issues the following advisory to a pilot flying on a heading of 360°:''TRAFFIC 10 O'CLOCK, 2 MILES, SOUTHBOUND...''Where should the pilot look for this traffic

Southwest.
Northwest.
Northeast.
3794[B]

An ATC radar facility issues the following advisory to a pilot during a local flight:''TRAFFIC 2 O'CLOCK, 5 MILES, NORTHBOUND...''Where should the pilot look for this traffic

Between directly behind and 90° to the right.
Between directly ahead and 90° to the right.
Between directly ahead and 90° to the left.
3795[C]

An ATC radar facility issues the following advisory to a pilot flying north in a calm wind:''TRAFFIC 9 O'CLOCK, 2 MILES, SOUTHBOUND...''Where should the pilot look for this traffic

South.
North.
West.
3796[A]

Basic radar service in the terminal radar program is best described as

traffic advisories and limited vectoring to VFR aircraft.
wind-shear warning at participating airports.
mandatory radar service provided by the Automated Radar Terminal System (ARTS) program.
3797[B]

From whom should a departing VFR aircraft request Stage II Terminal Radar Advisory Service during ground operations

Clearance delivery.
Ground control, on initial contact.
Tower, just before takeoff.
3798[C]

Stage III Service in the terminal radar program provides

warning to pilots when their aircraft are in unsafe proximity to terrain, obstructions, or other aircraft.
IFR separation (1,000 feet vertical and 3 miles lateral) between all aircraft.
sequencing and separation for participating VFR aircraft.
3799[C]

Which initial action should a pilot take prior to entering Class C airspace

Contact the FSS for traffic advisories.
Contact the tower and request permission to enter.
Contact approach control on the appropriate frequency.
3800[A]

When making routine transponder code changes, pilots should avoid inadvertent selection of which codes

7500, 7600, 7700.
1200, 1500, 7000.
0700, 1700, 7000.
3801[A]

When operating under VFR below 18,000 feet MSL, unless otherwise authorized, what transponder code should be selected

1200.
7600.
7700.
3802[A]

Unless otherwise authorized, if flying a transponder equipped aircraft, a recreational pilot should squawk which VFR code

1200.
7700.
7600.
3803[B]

If Air Traffic Control advises that radar service is terminated when the pilot is departing Class C airspace, the transponder should be set to code

0000.
1200.
4096.
3804[A]

If the aircraft's radio fails, what is the recommended procedure when landing at a controlled airport

Observe the traffic flow, enter the pattern, and look for a light signal from the tower.
Flash the landing lights and cycle the landing gear while circling the airport.
Enter a crosswind leg and rock the wings.
3805[C]

(Refer to figure 50.) Select the proper traffic pattern and runway for landing.

Left-hand traffic and Runway 22.
Left-hand traffic and Runway 18.
Right-hand traffic and Runway 18.
3806[B]

(Refer to figure 50.) If the wind is as shown by the landing direction indicator, the pilot should land on

Runway 36 and expect a crosswind from the right.
Runway 18 and expect a crosswind from the right.
Runway 22 directly into the wind.
3807[A]

(Refer to figure 51.) The segmented circle indicates that the airport traffic is

left-hand for Runway 36 and right-hand for Runway 18.
right-hand for Runway 9 and left-hand for Runway 27.
left-hand for Runway 18 and right-hand for Runway 36.
3808[C]

(Refer to figure 51.) The traffic patterns indicated in the segmented circle have been arranged to avoid flights over an area to the

north of the airport.
south of the airport.
southeast of the airport.
3809[A]

(Refer to figure 51.) The segmented circle indicates that a landing on Runway 26 will be with a

right-quartering headwind.
left-quartering headwind.
right-quartering tailwind.
3810[B]

(Refer to figure 51.) Which runway and traffic pattern should be used as indicated by the wind cone in the segmented circle

Right-hand traffic on Runway 9.
Left-hand traffic on Runway 36.
Right-hand traffic on Runway 18.
3811[C]

After landing at a tower-controlled airport, when should the pilot contact ground control

Prior to turning off the runway.
After reaching a taxiway that leads directly to the parking area.
When advised by the tower to do so.
3812[B]

If instructed by ground control to taxi to Runway 9, the pilot may proceed

to the next intersecting runway where further clearance is required.
via taxiways and across runways to, but not onto, Runway 9.
via taxiways and across runways to Runway 9, where an immediate takeoff may be made.
3813[A]

What ATC facility should the pilot contact to receive a special VFR departure clearance in Class D airspace

Air Traffic Control Tower.
Automated Flight Service Station.
Air Route Traffic Control Center.
3814[A]

What procedure is recommended when climbing or descending VFR on an airway

Execute gentle banks, left and right for continuous visual scanning of the airspace.
Advise the nearest FSS of the altitude changes.
Fly away from the centerline of the airway before changing altitude.
3815[B]

(Refer to figure 52.) If more than one cruising altitude is intended, which should be entered in block 7 of the flight plan

Lowest cruising altitude.
Initial cruising altitude.
Highest cruising altitude.
3816[C]

(Refer to figure 52.) What information should be entered in block 9 for a VFR day flight

The name of the airport of first intended landing.
The name of the airport where the aircraft is based.
The name of destination airport if no stopover for more than 1 hour is anticipated.
3817[C]

(Refer to figure 52.) What information should be entered in block 12 for a VFR day flight

The estimated time en route plus 45 minutes.
The estimated time en route plus 30 minutes.
The amount of usable fuel on board expressed in time.
3818[B]

How should a VFR flight plan be closed at the completion of the flight at a controlled airport

The tower will automatically close the flight plan when the aircraft turns off the runway.
The pilot must close the flight plan with the nearest FSS or other FAA facility upon landing.
The tower will relay the instructions to the nearest FSS when the aircraft contacts the tower for landing.
3819[B]

When activated, an emergency locator transmitter (ELT) transmits on

123.0 and 119.0 MHz.
121.5 and 243.0 MHz.
118.0 and 118.8 MHz.
3820[A]

When must the battery in an emergency locator transmitter (ELT) be replaced (or recharged if the battery is rechargeable)

After one-half the battery's useful life.
During each annual and 100-hour inspection.
Every 24 calendar months.
3821[A]

When may an emergency locator transmitter (ELT) be tested

During the first 5 minutes after the hour.
At 15 and 45 minutes past the hour.
Anytime.
3822[B]

Which procedure is recommended to ensure that the emergency locator transmitter (ELT) has not been activated

Ask the airport tower if they are receiving an ELT signal.
Monitor 121.5 before engine shutdown.
Turn off the aircraft ELT after landing.
3823[B]

Below FL180, en route weather advisories should be obtained from an FSS on

122.1 MHz.
122.0 MHz.
123.6 MHz.
3824[A]

Wingtip vortices are created only when an aircraft is

developing lift.
operating at high airspeeds.
heavily loaded.
3825[A]

The greatest vortex strength occurs when the generating aircraft is

heavy, clean, and slow.
heavy, dirty, and fast.
light, dirty, and fast.
3826[A]

Wingtip vortices created by large aircraft tend to

sink below the aircraft generating turbulence.
rise into the takeoff or landing path of a crossing runway.
rise into the traffic pattern.
3827[C]

When taking off or landing at an airport where heavy aircraft are operating, one should be particularly alert to the hazards of wingtip vortices because this turbulence tends to

rise from a crossing runway into the takeoff or landing path.
rise into the traffic pattern area surrounding the airport.
sink into the flightpath of aircraft operating below the aircraft generating the turbulence.
3828[C]

The wind condition that requires maximum caution when avoiding wake turbulence on landing is a

strong headwind.
light, quartering headwind.
light, quartering tailwind.
3829[B]

When landing behind a large aircraft, the pilot should avoid wake turbulence by staying

above the large aircraft's final approach path and landing before the large aircraft's touchdown point.
above the large aircraft's final approach path and landing beyond the large aircraft's touchdown point.
below the large aircraft's final approach path and landing before the large aircraft's touchdown point.
3830[A]

When departing behind a heavy aircraft, the pilot should avoid wake turbulence by maneuvering the aircraft

above and upwind from the heavy aircraft.
below and upwind from the heavy aircraft.
below and downwind from the heavy aircraft.
3831[C]

Pilots flying over a national wildlife refuge are requested to fly no lower than

3,000 feet AGL.
1,000 feet AGL.
2,000 feet AGL.
3832[A]

Large accumulations of carbon monoxide in the human body result in

loss of muscular power.
an increased sense of well-being.
tightness across the forehead.
3833[B]

What effect does haze have on the ability to see traffic or terrain features during flight

Haze causes the eyes to focus at infinity.
All traffic or terrain features appear to be farther away than their actual distance.
The eyes tend to overwork in haze and do not detect relative movement easily.
3834[C]

The most effective method of scanning for other aircraft for collision avoidance during daylight hours is to use

regularly spaced concentration on the 3-, 9-, and 12-o'clock positions.
peripheral vision by scanning small sectors and utilizing offcenter viewing.
a series of short, regularly spaced eye movements to search each 10-degree sector.
3835[B]

Which technique should a pilot use to scan for traffic to the right and left during straight-and-level flight

Continuous sweeping of the windshield from right to left.
Systematically focus on different segments of the sky for short intervals.
Concentrate on relative movement detected in the peripheral vision area.
3836[B]

How can you determine if another aircraft is on a collision course with your aircraft

The other aircraft will always appear to get larger and closer at a rapid rate.
There will be no apparent relative motion between your aircraft and the other aircraft.
The nose of each aircraft is pointed at the same point in space.
3837[C]

An ATC clearance provides
priority over all other traffic.
adequate separation from all traffic.
authorization to proceed under specified traffic conditions in controlled airspace.
3838[A]

(Refer to figure 53.) When approaching Lincoln Municipal from the west at noon for the purpose of landing, initial communications should be with

Lincoln Approach Control on 124.0 MHz.
Minneapolis Center on 128.75 MHz.
Lincoln Tower on 118.5 MHz.
3839[B]

(Refer to figure 53.) Which type radar service is provided to VFR aircraft at Lincoln Municipal

Sequencing to the primary Class C airport and standard separation.
Sequencing to the primary Class C airport, traffic advisories, conflict resolution, and safety alerts.
Sequencing to the primary Class C airport and conflict resolution so that radar targets do not touch, or 1,000 feet vertical separation.
3840[C]

(Refer to figure 53.) What is the recommended communications procedure for landing at Lincoln Municipal during the hours when the tower is not in operation

Monitor ATIS for airport conditions, then announce your position on 122.95 MHz.
Contact UNICOM on 122.95 MHz for traffic advisories.
Monitor airport traffic and announce your position and intentions on 118.5 MHz.
3841[B]

(Refer to figure 53.) Where is Loup City Municipal located with relation to the city

Northeast approximately 3 miles.
Northwest approximately 1 mile.
East approximately 10 miles.
3842[C]

(Refer to figure 53.) Traffic patterns in effect at Lincoln Municipal are

to the right on Runway 17L and Runway 35L; to the left on Runway 17R and Runway 35R.
to the right on Runways 14 - 32.
to the left on Runway 17L and Runway 35L; to the right on Runway 17R and Runway 35R.
3843[B]

The letters VHF/DF appearing in the Airport/Facility Directory for a certain airport indicate that

this airport is designated as an airport of entry.
the Flight Service Station has equipment with which to determine your direction from the station.
this airport has a direct-line phone to the Flight Service Station.
3844[B]

Which statement best defines hypoxia

An abnormal increase in the volume of air breathed.
A state of oxygen deficiency in the body.
A condition of gas bubble formation around the joints or muscles.
3845[B]

Rapid or extra deep breathing while using oxygen can cause a condition known as

aerosinusitis.
hyperventilation.
aerotitis.
3846[C]

Which would most likely result in hyperventilation

The excessive consumption of alcohol.
An extremely slow rate of breathing and insufficient oxygen.
Emotional tension, anxiety, or fear.
3847[C]

A pilot should be able to overcome the symptoms or avoid future occurrences of hyperventilation by

closely monitoring the flight instruments to control the airplane.
increasing the breathing rate in order to increase lung ventilation.
slowing the breathing rate, breathing into a bag, or talking aloud.
3848[B]

Susceptibility to carbon monoxide poisoning increases as

altitude decreases.
altitude increases.
air pressure increases.
3849[B]

What preparation should a pilot make to adapt the eyes for night flying

Avoid red lights at least 30 minutes before the flight.
Avoid bright white lights at least 30 minutes before the flight.
Wear sunglasses after sunset until ready for flight.
3850[B]

The danger of spatial disorientation during flight in poor visual conditions may be reduced by

shifting the eyes quickly between the exterior visual field and the instrument panel.
having faith in the instruments rather than taking a chance on the sensory organs.
leaning the body in the opposite direction of the motion of the aircraft.
3851[B]

A state of temporary confusion resulting from misleading information being sent to the brain by various sensory organs is defined as

hypoxia.
spatial disorientation.
hyperventilation.
3852[C]

Pilots are more subject to spatial disorientation if

eyes are moved often in the process of cross-checking the flight instruments.
they ignore the sensations of muscles and inner ear.
body signals are used to interpret flight attitude.
3853[C]

If a pilot experiences spatial disorientation during flight in a restricted visibility condition, the best way to overcome the effect is to

concentrate on yaw, pitch, and roll sensations.
consciously slow the breathing rate until symptoms clear and then resume normal breathing rate.
rely upon the aircraft instrument indications.
3854[C]

FAA advisory circulars containing subject matter specifically related to Airmen are issued under which subject number

70.
90.
60.
3855[B]

FAA advisory circulars containing subject matter specifically related to Airspace are issued under which subject number?

60.
70.
90.

الرئيسية || المنتدى || موسوعة المعرفة || اتصل بنا || خارطة الموقع

© Flying Way Copyright 2004-2010 - All rights reserved    Advertising with us